teorÍa de nÚmeros1 principios. problemas sin teoría. la "teoría de números" o...

133
TEORÍA DE NÚMEROS Enfoque Problem-solving Gerard Romo Garrido

Upload: others

Post on 13-Mar-2020

42 views

Category:

Documents


0 download

TRANSCRIPT

Page 1: TEORÍA DE NÚMEROS1 Principios. Problemas sin teoría. La "Teoría de números" o "aritmética" estudia las propiedades de los números enteros. Los conceptos teóricos de esta rama

TEORÍA DE NÚMEROS

Enfoque Problem-solving

Gerard Romo Garrido

Page 2: TEORÍA DE NÚMEROS1 Principios. Problemas sin teoría. La "Teoría de números" o "aritmética" estudia las propiedades de los números enteros. Los conceptos teóricos de esta rama

Toomates Cool·lección Los documentos de Toomates son materiales digitales y gratuitos. Son digitales porque están pensados para ser consultados mediante un ordenador, tablet o móvil. Son gratuitos porque se ofrecen a la comunidad educativa sin coste alguno. Los libros de texto pueden ser digitales o

en papel, gratuitos o en venta, y ninguna de estas opciones es necesariamente mejor o peor que las otras. Es más: Suele suceder que los mejores

docentes son los que piden a sus alumnos la compra de un libro de texto en papel, esto es un hecho. Lo que no es aceptable, por inmoral y mezquino, es el modelo de las llamadas "licencias digitales" con las que las editoriales pretenden cobrar

a los estudiantes, una y otra vez, por acceder a los mismos contenidos (unos contenidos que, además, son de una bajísima calidad). Este modelo

de negocio es miserable, pues impide el compartir un mismo libro, incluso entre dos hermanos, pretende convertir a los estudiantes en un mercado cautivo, exige a los estudiantes y a las escuelas costosísimas líneas de Internet, pretende pervertir el conocimiento, que es algo social,

público, convirtiéndolo en un producto de propiedad privada, accesible solo a aquellos que se lo puedan permitir, y solo de una manera encapsulada, fragmentada, impidiendo el derecho del alumno de poseer todo el libro, de acceder a todo el libro, de moverse libremente por todo

el libro.

Nadie puede pretender ser neutral ante esto: Mirar para otro lado y aceptar el modelo de licencias digitales es admitir un mundo más injusto, es participar en la denegación del acceso al conocimiento a aquellos que no disponen de medios económicos, en un mundo en el que las modernas

tecnologías actuales permiten, por primera vez en la historia de la Humanidad, poder compartir el conocimiento sin coste alguno, con algo tan

simple como es un archivo "pdf".

El conocimiento no es una mercancía.

El proyecto Toomates tiene como objetivo la promoción y difusión entre el profesorado y el colectivo de estudiantes de unos materiales didácticos libres, gratuitos y de calidad, que fuerce a las editoriales a competir ofreciendo alternativas de pago atractivas aumentando la calidad

de unos libros de texto que actualmente son muy mediocres, y no mediante retorcidas técnicas comerciales.

Este documento se comparte bajo una licencia “Creative Commons”: Se permite, se promueve y se fomenta cualquier uso, reproducción y

edición de todos estos materiales siempre que sea sin ánimo de lucro y se cite su procedencia. Todos los documentos se ofrecen en dos

versiones: En formato “pdf” para una cómoda lectura y en el formato “doc” de MSWord para permitir y facilitar su edición y generar versiones parcial o totalmente modificadas. Se agradecerá cualquier observación, comentario o colaboración a

[email protected]

Actualmente, Toomates Cool·lección consta de los siguientes libros:

Geometría axiomática:

GA Geometría Axiomática pdf 1 2 ... 23 portada

PG Problemas de Geometría pdf 1 2 3 4 5 6 7

Problem-solving:

AR Teoría de números pdf 1 2

PT Trigonometría pdf doc

DE Desigualdades pdf doc

PC Números complejos pdf doc

PA Álgebra (en preparación)

pdf doc

PC Combinatoria (en preparación)

pdf doc

PR Probabilidad (en preparación)

pdf doc

Libros de texto (En catalán)

AG Àlgebra pdf 1 2

GN Geometria analítica pdf doc

TR Trigonometria

pdf doc

CO Nombres complexos pdf doc

AL Àlgebra Lineal 2n batxillerat

pdf doc

GL Geometria Lineal 2n batxillerat

pdf doc

CI Càlcul Infinitesimal 2n batxillerat

pdf 1 2

PL Programació Lineal 2n batxillerat

pdf doc

Recopilaciones de problemas

SE Compendium OME 2005-2019 pdf

SA Compendium AIME 1983-2019 pdf

ST Compendium PAU TEC 1998-2019 pdf

SC Compendium PAU CCSS 1998-2019 pdf

PM Problemas de Matemáticas pdf doc

Versión de este documento: 11/03/2020

www.toomates.net

Page 3: TEORÍA DE NÚMEROS1 Principios. Problemas sin teoría. La "Teoría de números" o "aritmética" estudia las propiedades de los números enteros. Los conceptos teóricos de esta rama

Índice

1 Principios. →

El principio de buena ordenación. El principio de inducción. Principio del casillero.

2 Divisibilidad. → El algoritmo de la división.

3 Máximo común divisor. → Números coprimos. El Teorema de Bezout. El algoritmo de Euclides.

4 Divisibilidad con identidades algebraicas. →

5 Números primos. → Lema de Euclides. El Teorema fundamental de la aritmética (TFA).

6 Equivalencia modular. → Congruencias.

7 Introducción a las ecuaciones diofánticas. →

8 Ecuaciones diofánticas lineales. →

9 Congruencias lineales y sistemas de congruencias lineales. → Congruencias lineales. Inversos modulares. El Teorema chino del residuo.

10 El pequeño teorema de Fermat. →

11 La función Phi de Euler. → Teorema de Euler.

12 Orden de un entero. →

13 Números factoriales. → La función suelo. Números factoriales. La fórmula de Polignac.

14 Números combinatorios. Binomio de Newton. →

15 Números y primos de Fermat y de Mersenne. →

16 Número de divisores de un entero. → El producto de los divisores de un entero.

17 Suma de los divisores de un entero. → Números perfectos.

Soluciones. →

Fuentes. →

Apéndice. → El "problem-solving", tal y como yo lo entiendo.

Las competiciones AMC, un excelente sendero hacia las IMO detrás de un mar de siglas.

Page 4: TEORÍA DE NÚMEROS1 Principios. Problemas sin teoría. La "Teoría de números" o "aritmética" estudia las propiedades de los números enteros. Los conceptos teóricos de esta rama

1 Principios.

Problemas sin teoría.

La "Teoría de números" o "aritmética" estudia las propiedades de los números enteros.

Los conceptos teóricos de esta rama de las matemáticas pueden ser complicados, muy

complicados y terriblemente complicados. Sin embargo, muchos problemas se

resuelven con solo utilizar el sentido común, toda una serie de estrategias y conceptos

que, de tan obvios que son, los libros de teoría no dedican tiempo a explicarlos.

En este primer apartado se incluyen problemas cuya resolución no necesita ningún

concepto teórico previo, sólo el sentido común, la pura lógica, y algunas formulitas de

la matemática elemental. Sin embargo, no hay que despreciarlos. Es fundamental que el

estudiante dedique a cada problema tanto tiempo como sea necesario, y si no llega a

resolverlo, estudie detenidamente la solución que se presenta al final del libro.

Dedicar tiempo a pensar un problema y estudiar detenidamente la solución es la

base del aprendizaje "problem-solving".

Bases de numeración.

Diremos que n se escribe como 0121 ... aaaaa nnn en base 2b si

01

2

2

1

1 ... ababababan n

n

n

n

n

n

con bai 0 , INai , 0na

Por ejemplo, 3562 en base 7 es el número 13182767573 23 , y se escribe

73562 .

1.1 MF

¿Cuál de los siguientes enteros se puede expresar como la suma de 100 enteros positivos

consecutivos?

(A) 1,627,384,950 (B) 2,345,678,910 (C) 3,579,111,300 (D) 4,692,581,470 (E) 5,815,937,260

ASHME 1997 #20

1.2 M

Consideremos el entero cifras

N321

99...99..9999999999

Calcula la suma de todas las cifras de N.

AIME I 2019 #1

1.3 M

Para cada entero positivo n , sea nd la cifra de las unidades de n ..321 .

Determina el residuo cuando

2017

1n

nd se divide entre 1000.

AIME I 2017 #3

Page 5: TEORÍA DE NÚMEROS1 Principios. Problemas sin teoría. La "Teoría de números" o "aritmética" estudia las propiedades de los números enteros. Los conceptos teóricos de esta rama

1.4 MF

Sea 20 a , 51 a , 82 a , y para cada 2n , define na recursivamente como el

residuo cuando 3214 nnn aaa se divide entre 11. Determina 202220202018 aaa .

AIME II 2018 #2

1.5 F

Multiplicamos todos los números pares del 2 al 98 inclusive, excepto aquellos acabados

en 0. ¿Cuál será la cifra de las unidades del resultado?

(A) 0 (B) 2 (C) 4 (D) 6 (E) 8

AMC10 1999 Sample #14

1.6 M

Demostrar que si entre los infinitos términos de una progresión aritmética de números

enteros hay un cuadrado perfecto, entonces infinitos términos de la progresión son

cuadrados perfectos.

OME 1993-94 (Primera sesión) #1

Principio de la buena ordenación.

Todo conjunto S de números enteros no negativos contiene un elemento mínimo, es

decir, existe un elemento Sa tal que ba para todo Sb .

Teorema. Propiedad arquimediana de los números naturales.

Si a y b son números enteros positivos, entonces existe un entero positivo n tal que

ban .

Demostración.

Supongamos que no es cierto, es decir, que existen dos números 0, ba tales que

ban para todo 0n .

Consideremos el conjunto 0, nanbS

Está claro que es un subconjunto de números enteros positivos, pues

nabban 0 , y por tanto le podemos aplicar el Principio de la buena

ordenación, es decir, contendrá un elemento mínimo amb , para cierto 0m .

Pero, por hipótesis, amb )1( también pertenecerá a S, luego:

mabamabamb )1( , pues 1a , luego amb no puede ser el mínimo,

llegando a contradicción.

Así pues, la propiedad arquimediana de los números naturales debe ser cierta, pues su

negación nos lleva a contradicción.

Page 6: TEORÍA DE NÚMEROS1 Principios. Problemas sin teoría. La "Teoría de números" o "aritmética" estudia las propiedades de los números enteros. Los conceptos teóricos de esta rama

Principio de Inducción.

Sea S un conjunto de números enteros positivos cumpliendo las dos condiciones

siguientes:

a) 1 pertenece a S.

b) Si Sn , entonces Sn 1

Entonces S es el conjunto de todos los enteros positivos: ...,3,2,1S

Demostración.

Sea ST ...,3,2,1 , y supongamos que T , es decir, que no está vacío, o lo que

es lo mismo, que no se cumple el Principio de Inducción.

Aplicando el Principio de la buena ordenación, T contendrá elemento mínimo,

llamémosle a .

Puesto que T1 , pues por hipótesis, S1 , está claro que 1a , luego aa 10 .

El número 1a tampoco pertenecerá a S, pues si SaaSa 111 ,

contradiciendo la hipótesis. Pero aa 1 , llegando a contradicción, pues habíamos

supuesto que a era mínimo.

El principio de inducción es una herramienta muy poderosa para demostrar fórmulas

que nos serán muy útiles para solucionar una enorme variedad de problemas.

Ejemplo 1.

6

)1)(12(...321 2222

nnn

n para todo ...,3,2,1n

Demostración.

Sea S el conjunto de números enteros positivos para los que la fórmula anterior es

cierta.

Está claro que 1 pertenece a S, pues

6

)11)(112(1112

Supongamos que la fórmula anterior se cumple para un cierto valor n , y veamos que,

entonces, se cumplirá también para 1n :

(*)6

)1(6)12()1(

)1(6

)12()1()1(

6

)1)(12(1...321 222222

nnnn

nnn

nnnnn

nn

)32)(2(672662)1(6)12( 22 nnnnnnnnnn

6

)11)(1)1(2)(1(

6

)32)(2)(1(

6

)32)(2()1((*)

nnnnnnnnn

6

)1)(12(

kkk, tomando 1 nk , luego la fórmula también es válida para 1n .

Así pues, ...,3,2,1S , es decir, la fórmula es válida para todos los enteros positivos.

Page 7: TEORÍA DE NÚMEROS1 Principios. Problemas sin teoría. La "Teoría de números" o "aritmética" estudia las propiedades de los números enteros. Los conceptos teóricos de esta rama

Ejercicios.

Demuestra por inducción las fórmulas siguientes (para todo ...,3,2,1n ) :

a) 2

)1(...321

nnn

b) 2)12(...531 nn

c) 3

)2)(1()1(...433221

nnnnn

d) 3

)12()12()12(...531 2222

nnnn

e)

2

3333

2

)1(...321

nnn

f) r

rr

kk

n

n

1

1 1

0

(“Serie Geométrica”)

Principio del casillero.

La idea que subyace en este principio es muy sencilla: Si tenemos tres automóviles y

solo dos garajes, necesariamente en uno de los garajes habrá más de un automóvil. El

enunciado general es el siguiente:

Si debemos distribuir 1n objetos en n celdas o casillas, entonces al menos una de

ellas contiene más de un objeto.

Este principio se conoce también como Principio del palomar o Principio de Dirichlet

(Dirichlet, P.G. Lejeune 1805-1859).

1.7 F

¿Cuántas personas hay que reunir para asegurar que hay al menos dos que tengan

nombres con la misma inicial?

1.8 F

¿Cuántas personas hay que reunir para asegurar que hay al menos seis que tengan

nombres con la misma inicial?

1.9 F

Demostrar que en un grupo de siete personas hay como mínimo 4 con el mismo sexo.

1.10 F

Sea A un conjunto de veinte enteros tomados de la progresión aritmética

100,...,12,8,4,1 .

Demostrar que existen en A dos enteros diferentes cuya suma es 104.

PUTNAM 1978

Page 8: TEORÍA DE NÚMEROS1 Principios. Problemas sin teoría. La "Teoría de números" o "aritmética" estudia las propiedades de los números enteros. Los conceptos teóricos de esta rama

1.11 F

Demuestra que, en todo conjunto de siete números positivos distintos no superiores a

126, se encuentran dos elementos ba , tales que aba 2 .

1.12 F

Demostrar que en todo subconjunto de 55 elementos de 100,...,3,2,1 siempre

podemos encontrar dos elementos que cuya diferencia sea 10.

Page 9: TEORÍA DE NÚMEROS1 Principios. Problemas sin teoría. La "Teoría de números" o "aritmética" estudia las propiedades de los números enteros. Los conceptos teóricos de esta rama

2 Divisibilidad.

Armados únicamente con los conceptos más básicos de la divisibilidad, los que se aprenden a

los doce años, ya podemos enfrentarnos a problemas muy interesantes, incluso de nivel AIME.

Algoritmo de la división.

Para todo Za y Nb , existen Zq y br 0 únicos, llamados respectivamente

cociente y residuo de la división, tales que

rbqa

Ejemplo.

Aplicando el Algoritmo de la división, demuestra que todo cuadrado es siempre de la

forma k4 o 14 k .

Aplicando el Algoritmo de la división, todo número n será de la forma

1)1(414434,24,14,4 bbbnbnbnbn .

Veamos su cuadrado, caso por caso:

)144(4416162424

1)24(418161414

4444

2222

2222

222

bbbbbnbn

bbbbbnbn

bbnbn

Sea cual sea el caso, siempre es de la forma k4 o 14 k .

Divisibilidad. Dados dos números enteros ba, , diremos que a divide a b, o que b es divisible entre a,

y escribiremos b|a , cuando exista un tercer número entero c tal que bca , es decir,

cuando al realizar la división entera b entre a el residuo sea cero.

Propiedades de la divisibilidad.

a) aa | para todo entero a (propiedad reflexiva)

b) ba | y c|b c|a (propiedad transitiva)

c) cybxaayba |c|| para cualquier par de enteros yx,

d) bd|d|| accyba . En particular, cbaba || .

e) ba | y cacba ||

Criterios de divisibilidad.

Entre 2: Cuando acaba en cero o cifra par.

Entre 3: Cuando la suma de sus cifras es múltiplo de 3.

Entre 5: Cuando acaba en 0 o en 5.

Entre 11: Cuando la diferencia entre la suma de sus cifras pares y la suma de sus cifras

impares sea 0 o múltiplo de 11.

Page 10: TEORÍA DE NÚMEROS1 Principios. Problemas sin teoría. La "Teoría de números" o "aritmética" estudia las propiedades de los números enteros. Los conceptos teóricos de esta rama

2.1 MF

El número de dígitos de 251654 (cuando está escrito en la base 10 usual) es

(A) 31 (B) 30 (C) 29 (D) 28 (E) 27

AHSME 1984 #9

2.2 F

Demuestra que el cuadrado de un número impar es siempre de la forma 18 k

OPOS BALEARES 2018

2.3 F

El perímetro de un triángulo equilátero excede el perímetro de un cuadrado en 1989 cm.

La longitud de cada lado del triángulo excede la longitud de cada lado del cuadrado en d

cm. El cuadrado tiene perímetro mayor que 0. ¿Cuántos posibles enteros positivos no

son válidos para d?

(A) 0 (B) 9 (C) 221 (D) 663 (E) infinitos

ASHME 40 #17

2.4 F

¿Cuántos números en base 10, dcbaN satisfacen todas las tres condiciones

siguientes?

(i) 60004000 N

(ii) N es múltiplo de 5

(iii) 63 cb

(A) 10 (B) 18 (C) 24 (D) 36 (E) 48

AHSME 1995 #12

2.5 F

La profesora Walter corrige un examen de matemáticas de sus cinco alumnos. Entra en

orden aleatorio las puntuaciones en una hoja de cálculo, que va recalculando la media

de la clase después de cada puntuación (sobre el número de alumnos ya introducidos, no

sobre el total de 5). La profesora se da cuenta de que, después de cada puntuación, la

media es siempre un entero. Las puntuaciones (presentadas en orden ascendente) son

71, 76, 80, 82 y 91. ¿Cuál fue la última puntuación que introdujo la profesora Walter?

(A) 71 (B) 76 (C) 80 (D) 82 (E) 91

AMC12 2000 #9

2.6 F

Determina los valores enteros de x para los cuales la expresión xx 62 es el cuadrado

de un entero.

Page 11: TEORÍA DE NÚMEROS1 Principios. Problemas sin teoría. La "Teoría de números" o "aritmética" estudia las propiedades de los números enteros. Los conceptos teóricos de esta rama

2.7 M

Determina la suma de todos los números enteros positivos 1000b tal que el número

b36 (escrito en base b) es un cuadrado perfecto y el número b27 (también escrito en

base b) es un cubo perfecto.

AIME II 2018 #3

2.8 F

Determina la suma de todos los enteros positivos n tales que 2017852 nn es un

entero.

AIME II 2017 #6

2.9 F

Demuestra que ningún número de la forma ...,11111,1111,111,11 es un cuadrado

perfecto.

Indicación: Todo número de la forma 111...111 se puede escribir como

343108...111111...111 k .

2.10 MF

¿Para cuantos enteros n entre 1 y 100 el polinomio nxx 2 factoriza en el producto

de dos factores lineales con coeficientes enteros?

(A) 0 (B) 1 (C) 2 (D) 9 (E) 10

ASHME 1989 #8

2.11 F

Sea S el número de pares ordenados de enteros ),( ba con 1001 a y 0b tales que

el polinomio baxx 2 pueda ser factorizado como producto de dos (no

necesariamente distintos) factores lineales con coeficientes enteros. Determina el

residuo cuando S se divide entre 1000.

AIME I 2018 #1

2.12 F

Una sucesión pucelana es una sucesión crecientes de dieciséis números impares

positivos consecutivos, cuya suma es un cubo perfecto. ¿Cuántas sucesiones pucelanas

tienen solamente números de tres cifras?

OME Fase Nacional 2010 #1

2.13 M

a) Demuestra que el producto de dos números consecutivos es par.

b) Demuestra que el producto de tres números consecutivos es divisible entre 6.

c) Demuestra que nn 5 es divisible entre 30.

Page 12: TEORÍA DE NÚMEROS1 Principios. Problemas sin teoría. La "Teoría de números" o "aritmética" estudia las propiedades de los números enteros. Los conceptos teóricos de esta rama

2.14 F

Determina la suma de todos los números primos entre 1 y 100 tal que sean

simultáneamente 1 mayor que un múltiplo de 4 y 1 menor que un múltiplo de 5

ASHME 1999 #4

2.15 MF

¿Cuántos enteros positivos b existen con la propiedad de que 729logb sea un entero

positivo?

(A) 0 (B) 1 (C) 2 (D) 3 (E) 4

AMC12 2000 #7

2.16 F

Aplicando el Algoritmo de la división, demuestra que:

a) El cuadrado de todo entero es siempre de la forma k3 o 13 k .

b) Un número de la forma 13 2 a nunca puede ser el cuadrado de un entero.

2.17 F

¿Para cuántos enteros N entre 1 y 1990 la fracción impropia 4

72

N

N no es irreducible?

ASHME 1990 #19

2.18 MF

Sea r el residuo cuando 1059, 1417 y 2312 se dividen entre 1d . Determina el valor de

rd .

AHSME 1976

2.19 F

Demuestra que existen infinitos números enteros n tales que 232 n es divisible por 24.

2.20 M

Determina todos los enteros positivos d tales que d divide 12 n y 1)1( 2 n para

algún entero n.

Page 13: TEORÍA DE NÚMEROS1 Principios. Problemas sin teoría. La "Teoría de números" o "aritmética" estudia las propiedades de los números enteros. Los conceptos teóricos de esta rama

Divisibilidad y orden.

Divisibilidad implica desigualdad.

Si a y b son positivos, baba | .

En general, si ba | y 0b ba . En particular, 11| aa

2.21 F

Sean ndddd k ...1 321 los divisores del entero positivo n.

Encuentra todos los números n tales que 3

3

2

2 ddn .

México 2008

Distribución de los divisores de 2n .

Un dato que puede ser útil es que n es el divisor central de 2n , es decir, el número de

divisores de 2n menores que n es igual al número de divisores de 2n mayores que n.

2.22 M

Sea 1931 32n . Determina el número de enteros positivos de 2n que sean menores que

n y que no sean divisores de n .

AIME 1995 #6

Page 14: TEORÍA DE NÚMEROS1 Principios. Problemas sin teoría. La "Teoría de números" o "aritmética" estudia las propiedades de los números enteros. Los conceptos teóricos de esta rama

3 Máximo común divisor.

Máximo común divisor. Números coprimos. Mínimo común múltiplo.

Definimos el máximo común divisor de dos números ba, positivos como el mayor

divisor positivo común de ambos, y escribiremos ba, . La unidad siempre es divisor

común, por lo que siempre existe el máximo común divisor, y 1, ba . Diremos que

dos números son coprimos cuando su único divisor común es el 1, es decir, cuando

1, ba .

Definimos el mínimo común múltiplo de dos números ba, positivos como el menor

múltiplo común de ambos, y escribiremos ba, . Está claro que el producto ab es

múltiplo común de ambos, luego el mcm está bien definido.

Algunas propiedades del mcd y del mcm.

a) baba ,|, y baba ,|, .

b) bababa ,,, .

c) banbnan ,, , nnn baba ,, y nnn baba ,, .

d) baaba ,, , y bbaa |, .

e) cabacba ,,,,, y cabacba ,,,,, .

f) bababa ,, .

Interpretación geométrica del máximo común divisor.

El máximo común divisor de a y b indica el número de puntos ),( yx con coordenadas

enteras en el segmento que une los puntos )0,0( y ),( ba , sin contar el inicial )0,0( .

Por ejemplo, 3)6,15( , y por tanto el segmento que une los puntos )0,0( y )6,15(

pasa por tres puntos con coordenadas enteras, aparte del propio )0,0( :

)6,15()24,510(

)4,10()22,55(

)2,5(

23

6

53

15

3.1 F

Diremos que un punto ),( yx del plano es “punto entero” cuando sus coordenadas sean

enteras. ¿Cuántos de estos puntos de este tipo hay (incluyendo ambos extremos) en el

segmento que cuyos extremos son )17,3( y )281,48( ?

ASHME 1989 #16

Page 15: TEORÍA DE NÚMEROS1 Principios. Problemas sin teoría. La "Teoría de números" o "aritmética" estudia las propiedades de los números enteros. Los conceptos teóricos de esta rama

Lema.

),( ba divide a cualquier combinación lineal de a y b .

Demostración. Sea byax una combinación lineal de a y b .

byaxbabybaba

axbaaba

|),(

|),(b|),(

|),(|),(

Teorema de Bezout (TDB).

Dados dos números enteros ba, no ambos cero, el máximo común divisor de ba, se

caracteriza por ser el elemento mínimo del conjunto no vacío

0,,, byaxZyxbyaxA

Luego existe, es único y siempre se puede escribir como combinación lineal de a y b :

Existen enteros yx, tales que byaxba ),(

Demostración.

Consideremos el conjunto anterior 0,,, byaxZyxbyaxA .

Es un conjunto no vacío pues al menos 022 babbaa pertenece a A (estamos en

todo momento suponiendo que ba, no son ambos cero).

Por el Principio de buena ordenación, A tendrá un mínimo, al que llamaremos d .

Vamos a demostrar que ),( bad .

Supongamos que 011 ybxad para ciertos enteros 11, yx .

Por el Algoritmo de la división, existirán enteros rq, tales que rqda con

dr 0 .

Si 0r , entonces

qybqxaqybqxaaqybxaadqar 111111 )1()(0 , y por tanto, r

pertenece al conjunto A, tomando qyyqxx 1212 ,1 . Pero dr , lo cual

contradice la hipótesis de d como elemento mínimo. Luego 0r y por tanto qda , es

decir, d divide al número a.

Con el mismo razonamiento se demuestra que d divide al número b, y por tanto d es

común divisor de a y b. Veamos que es el máximo común divisor.

Sea m otro común divisor de a y b, entonces, aplicando el lema anterior,

dmdmdybxam || 11 .

Corolario.

Si nxx ,...,1 son números enteros y a es cualquier número entero positivo,

nn xxaxaxa ,...,,..., 11

Demostración.

Sean nxaxad ,...,1 y nxxe ,...,1 . Luego deadeaxaeaxe ii ||| .

Por el TDB, nnnn axkaxkeaxkxke ...... 1111

Page 16: TEORÍA DE NÚMEROS1 Principios. Problemas sin teoría. La "Teoría de números" o "aritmética" estudia las propiedades de los números enteros. Los conceptos teóricos de esta rama

Es decir, ae es combinación lineal de nxaxa ,...,1 , y por tanto es un múltiplo de d ,

luego dae . Y, puesto que anteriormente hemos demostrado que dea , llegamos a

dea .

Corolario.

1),( ba Existe una combinación lineal 1 byax

Demostración.

Es el TDB.

1),(1|),(1|),( bababyaxba

3.2 M

Demuestra que la fracción 314

421

n

n es irreducible para todo número natural n.

IMO 1959

3.3 M

Los números de la sucesión ...,116,109,104,101 son de la forma 2100 nan ,

...,2,1n

Para cada n , sea 1, nnn aad . Determina nn d1max .

AIME 1985

Teorema.

Si naaad ...,,, 21 entonces 1...,,, 21

d

a

d

a

d

a n

Demostración.

dkaadaaad iiin |...,,, 21 . Sea

d

a

d

a

d

ad n...,,,' 21 y supongamos que

1'd .

dddaddddkadkd

a

d

ad

d

a

d

a

d

ad iiii

iin |'|'''''|'...,,,' 21

Lo cual es imposible suponiendo 1'd . Luego 1'd .

Teorema.

Si cba ,, son tres números enteros, con 1),( ba , cabca || .

Demostración.

Por el TDB, byaxba 11),( para ciertos enteros yx, . Luego bcyacxc , y

puesto que trivialmente acxa | y por hipótesis bcya | , se deduce cbcyacxa | .

Page 17: TEORÍA DE NÚMEROS1 Principios. Problemas sin teoría. La "Teoría de números" o "aritmética" estudia las propiedades de los números enteros. Los conceptos teóricos de esta rama

El algoritmo de Euclides.

El algoritmo de Euclides es un método efectivo para calcular el máximo común divisor

de dos números a y b que se basa en el algoritmo de la división y el siguiente

principio:

Si ckab , entonces ),(),(),( cackaaba

Suponiendo ba , podemos dividir a entre b para expresar rkba , con br y

así

),(),(),( brbrbkba

Este mismo proceso repetiremos una y otra vez, con números más y más pequeños,

hasta que el máximo común divisor se haga evidente.

Ejemplo 1.

Calcula 23,29 mediante el Algoritmo de Euclides.

Solución.

)23,6()23,6231()23,29(623129

)6,5()6,563()6,23(56323

1)5,1()5,151()5,6(1516

Así pues, 123,29

Ejemplo 2.

Calcula 246,3456 mediante el Algoritmo de Euclides.

Solución.

)246,158()246,15824613()246,3456(

)88,158()881581,158()246,158(

Y de la misma manera: 2)16,2()18,16()70,18()88,70()88,158(

Luego 2246,3456

Page 18: TEORÍA DE NÚMEROS1 Principios. Problemas sin teoría. La "Teoría de números" o "aritmética" estudia las propiedades de los números enteros. Los conceptos teóricos de esta rama

4 Divisibilidad con identidades algebraicas.

Dos identidades algebraicas importantes.

La identidad algebraica

122321 ... nnnnnnn yyxyxyxxyxyx

es fundamental en la resolución de todo tipo de problemas de Aritmética.

Su demostración es fácil: Basta con desarrollar el producto de la derecha:

nn

nnnnnnnnnn

nnnnn

yx

yyxyxyxyxxyyxyxyxx

yyxyxyxxyx

133221122221

122321

......

...

En particular, tomando 1y obtenemos una identidad muy útil:

1

11...

121

x

xxxxx

nnnn

De la primera identidad se desprende directamente el siguiente resultado:

a) nn baba | para cualquier 1n .

Por ejemplo, sin necesidad de ningún cálculo, 23452345 81018767 es divisible entre 666.

Que se puede generalizar para obtener el siguiente resultado:

b) Si nd | con d y n positivos, entonces nndd baba | para todo entero ba, .

En efecto, supongamos que dkn para cierto entero k . Entonces

...2321 babaababababa dddddkdkddkdknn

Otra identidad algebraica muy útil es

Si n es impar, 122321 ... nnnnnnn yyxyxyxxyxyx

En efecto, Si n es impar, nnnn yxyx , y basta aplicar la identidad anterior.

De esta segunda identidad se desprende de forma directa el siguiente resultado:

c) Si n es impar, nn baba |

Que se puede generalizar para obtener el siguiente resultado:

e) Si nd | con d y n positivos, y dn / es impar, entonces nndd baba | para todo

entero ba, .

Page 19: TEORÍA DE NÚMEROS1 Principios. Problemas sin teoría. La "Teoría de números" o "aritmética" estudia las propiedades de los números enteros. Los conceptos teóricos de esta rama

Proposición.

Si 12 n es primo, entonces n es una potencia de 2.

Demostración.

Supongamos, por el contrario, que n no es una potencia de 2, es decir, que podemos

escribir hkn , con 1k impar.

Entonces

12...2221212121212321

hkhkhkhhkkhkhhkn

Y por tanto nuestro número es divisible entre 12 h .

Observación 1.

Este resultado será fundamental en el Tema 15, como base para definir los “primos de

Fermat”.

Observación 2.

El recíproco no es cierto. Por ejemplo: 12|641 32 .

En efecto, utilizando que 12552641 744 ,

16406392641

16406396416412116416412152522

1525221552212212

228

228428474428

42844284442842832

En donde hemos utilizado que:

16406416396416416406396411640640639

6416406406391640640640164016401640

222

2323444

(En el Tema 6 volveremos a demostrar este resultado aplicando la aritmética modular)

4.1 F

Determina todos los números primos de la forma 13 n , para enteros 1n .

4.2 M

Demostrar que nnnn 2614648032903 es divisible entre 1897 para todo natural n .

EOTVOS 1899

4.3 M

Demuestra que 44 n con INn es primo si y solo si 1n .

4.4 M

Determina todos los enteros 1n para los cuales nn 44 es un número primo.

Page 20: TEORÍA DE NÚMEROS1 Principios. Problemas sin teoría. La "Teoría de números" o "aritmética" estudia las propiedades de los números enteros. Los conceptos teóricos de esta rama

4.5 F

Demuestra que, para todo INn , 2n divide a 11 n

n .

4.6 F

Demostrar que 1001 divide 1993199319931993 1000...321

4.7 M

Demuestra que 7 divide al número 474747474747 654321

4.8 F

Si al cuadrado de un número de dos dígitos se le resta el cuadrado del número formado

invirtiendo el orden de sus dígitos, entonces el resultado no siempre será divisible por:

(A) 9 (B) El producto de los dígitos. (C) La suma de los dígitos.

(D) La diferencia de los dígitos. (E) 11

ASHME 1957 #24

Page 21: TEORÍA DE NÚMEROS1 Principios. Problemas sin teoría. La "Teoría de números" o "aritmética" estudia las propiedades de los números enteros. Los conceptos teóricos de esta rama

5 Números primos.

Números primos. Todo número es divisible por sí mismo y por la unidad. Diremos que un número natural

1p es primo cuando solo sea divisible por sí mismo y por la unidad. Por ejemplo, son

primos los números 5, 13, 59 o 397. Llamamos compuestos a los números que no son

primos. El número 1 no se considera ni primo ni compuesto.

Algunas propiedades de los números primos.

a) Si p es primo y pa | , entonces pa ,1 .

b) Si p y q son primos, entonces qpqp | .

c) Todos los primos son impares excepto el 2.

d) El 2 y el 3 son los únicos primos cuya diferencia es 1.

e) Si p es primo y Nba , , entonces apap b || .

Ejemplo.

Demuestra que si 3p es primo, entonces 1|24 2 p .

Solución.

Por el algoritmo de la división, todo número se puede representar como n6 , 16 n o

26 n .

Si es primo, la única opción aceptable es 16 n , pues las otras son divisibles entre 2 o

3.

Luego

)13(1212361112361616 22222 nnnnpnnnpnp

Está claro que o bien n es par o bien 13 n es par, luego 1|24 2 p .

Ejemplo.

Determina la suma de todos los números primos entre 1 y 100 que son simultáneamente

1 más que un múltiplo de 4 y 1 menos que un múltiplo de 5.

AHSME 1999 #3

Solución.

1101)2(52|25|2

12245151415

14

ccpcbbb

aabbabp

ap

Con 110 cp ya tenemos un conjunto de candidatos suficientemente pequeño como

para proceder a testearlos, uno por uno:

911101 pc (no es primo) 3441912102 pc

1742913103 pc 3943914104 pc

4915105 pc 31445916106 pc

6917107 pc 31947918108 pc

12248919109 pc 991101010 pc

Luego la suma es 29+89=118.

Page 22: TEORÍA DE NÚMEROS1 Principios. Problemas sin teoría. La "Teoría de números" o "aritmética" estudia las propiedades de los números enteros. Los conceptos teóricos de esta rama

Lema de Euclides.

Si p es primo y abp | entonces ap | o bp | .

Demostración.

Sea p un número primo. Si ap | entonces 1),( pa y por el TDB existirán enteros

yx, tales que pyax 1 , y por tanto bpybaxb .

Pero baxpabp || , y claramente bpyp | , luego bbpybaxp |

5.1 M

Sean yx, enteros. Demostrar que yx 32 es divisible entre 17 si y solo si yx 59 es

divisible entre 17.

Corolario.

a) a es par si y solo si na es par

b) a es impar si y solo si na es impar.

Demostración.

a) nnnn kkaka 122)2(2

Basta aplicar el Lema de Euclides con 2p .

b)

n

k

knn kk

nkaka

1

21)12(12 impar.

Por el apartado a, si a es par entonces na es par, luego si na es impar,

necesariamente a debe ser par.

Corolario.

Hay infinitos números primos.

Demostración.

Entre otras muchas demostraciones de este resultado, la de Euclides es un ejemplo de

elegancia:

Supongamos, por el contrario, que existe una cantidad finita de números primos. Sean

estos

nppp ...1 21

Consideremos el número 121 npppn .

No puede ser primo, pues npn , luego existirá al menos un k tal que npk | , pero

también se cumple trivialmente nk pppnp 211| , y por tanto

11|1)1(| kkk ppnnp , lo cual es imposible. Así pues, no es posible que

exista un número finito de primos.

Page 23: TEORÍA DE NÚMEROS1 Principios. Problemas sin teoría. La "Teoría de números" o "aritmética" estudia las propiedades de los números enteros. Los conceptos teóricos de esta rama

Teorema fundamental de la aritmética (TFA).

Todo entero positivo n se descompone de forma única como producto de potencias de

números primos (el orden es irrelevante). ka

k

aapppn ...21

21

La expresión anterior se denomina descomposición canónica de n .

Por ejemplo: 23218 , 73284 2 , 131124576 , 8179232716

Este teorema es la clave para resolver muchísimos problemas de aritmética, pues reduce

el problema a un recuento de casos.

Proposición.

Dado un número en descomposición canónica ka

k

aapppn ...21

21 ,

a) Todo divisor de n es de la forma ke

k

eepppd ...21

21 , con ii ae 0 .

b) El número de divisores de n es 1...11 21 kaaa .

Demostración.

a) Todo número de la forma ke

k

eepppd ...21

21 , con ii ae 0 es un divisor de n, y son

todos diferentes por el TFA.

b) Basta aplicar el principio fundamental del conteo.

5.2 F

Existen enteros positivos A, B y C, sin factores comunes mayores que 1, tales que

CBA 2log5log 200200

Determina CBA . ASHME 1995 #24

5.3 M

Demostrar que existe un único número natural n tal que n222 118 es un cuadrado

perfecto.

5.4 F

Si 1998 se escribe como producto de dos enteros positivos cuya diferencia es lo más

pequeña posible, entonces la diferencia es:

(A) 8 (B) 15 (C) 17 (D) 47 (E) 93

AHSME 1998 #6

5.5 M

Determina los tres números naturales consecutivos más pequeños cuya suma es un

cuadrado perfecto y un cubo perfecto de números naturales.

Page 24: TEORÍA DE NÚMEROS1 Principios. Problemas sin teoría. La "Teoría de números" o "aritmética" estudia las propiedades de los números enteros. Los conceptos teóricos de esta rama

5.6 M

Halla todas las sucesiones finitas de n números naturales consecutivos naaa ,...,, 21 , con

3n , tales que 2009,...21 naaa

OME Fase Nacional 2009 #1

5.7 MF

En el año 2001, los Estados Unidos acogieron las Olimpiadas Matemáticas. Sean

OMI ,, enteros positivos tales que 2001 OMI . ¿Cuál será el valor más grande

posible de la suma OMI ?

(A) 23 (B) 55 (C) 99 (D) 111 (E) 671

AMC12 2000 #1

5.8 F

Existen enteros positivos CBA ,, , sin factores comunes mayores que 1, tales que

CBA 2log5log 200200 .

¿Cuál es el valor de CBA ?

(A) 6 (B) 7 (C) 8 (D) 9 (E) 10

AHSME 1995 #24

5.9 M

¿Cuántos conjuntos de tres elementos cba ,, de enteros positivos verifican

2310 cba ?

(A) 32 (B) 36 (C) 40 (D) 43 (E) 45

AHSME 1995 #29

5.10 F

Existe un número primo p tal que 116 p es el cubo de un entero positivo. Determina p.

AIME I 2015 #3

5.11 F

Cuando los números 702, 787 y 855 son divididos entre el entero positivo m, el resto es

siempre el mismo entero positivo r. Cuando los números 412, 722 y 815 son divididos

entre el entero positivo n, el resto es siempre el mismo entero positivo rs . Determina

srnm .

AIME I 2017 #2

Page 25: TEORÍA DE NÚMEROS1 Principios. Problemas sin teoría. La "Teoría de números" o "aritmética" estudia las propiedades de los números enteros. Los conceptos teóricos de esta rama

5.12 M

Determina el número de polinomios de segundo grado )(xf con coeficientes enteros y

ceros enteros tales que 2010)0( f .

AIME II 2010 #10

5.13 M

Determina todos los primos p para los cuales la ecuación 04442 ppxx tiene

soluciones enteras.

ASHME 1987 #23

5.14 M

Demostrar que en cualquier conjunto de 1n números entre 1 y n2 siempre podemos

encontrar dos elementos tales que el menor divide al mayor.

5.15 F

Determina el número de valores de k sabiendo que 1212

es el mínimo común múltiplo de

66, 8

8 y k.

AIME 1998

5.16 F

Denotamos por [r,s] el mínimo común múltiplo de los enteros positivos r y s. Determina

el número de triplas ordenadas a, b, c tales que [a,b]=1000, [b,c]=2000 y [c,a]=2000.

5.17 F

Determina 223 yx si yx, son enteros tales que 517303 2222 xyxy

AIME 1987 #5

Page 26: TEORÍA DE NÚMEROS1 Principios. Problemas sin teoría. La "Teoría de números" o "aritmética" estudia las propiedades de los números enteros. Los conceptos teóricos de esta rama

6 Equivalencia modular.

El lenguaje de las congruencias nos permite abordar con éxito problemas aparentemente muy

difíciles. Las congruencias es un lenguaje, una técnica, y por tanto solo se aprende practicando,

jugando con ella durante mucho tiempo.

Definición de congruencias.

Diremos que )(mod nba , y diremos que "a es congruente con b módulo n" cuando

sucede alguna de estas condiciones equivalentes:

a) )(| ban

b) bnka para cierto entero k .

c) a y b dejan el mismo residuo cuando son divididos entre n.

Por ejemplo: )7(mod324 , pues 37324 , )5(mod434 , pues 45634

En particular: anna |)(mod0

Ejemplo.

Determina x tal que )8(mod65 x .

Solución.

Vamos probando, uno por uno:

)8(mod5580515 )8(mod22811025

)8(mod77811535 )8(mod44822045

)8(mod11832555

)8(mod66833065 La solución es )8(mod6x

)8(mod33843575

Propiedades de las congruencias.

Sea 0n fijo, y dcba ,,, enteros arbitrarios. Entonces se cumple:

a) )(mod naa (Propiedad reflexiva).

b) Si )(mod nba )(mod nab (Propiedad simétrica).

c) Si )(mod nba y )(mod ncb )(mod nca (Propiedad transitiva).

d) Si )(mod nba y )(mod ndc )(mod ndbca y )(mod ndbca .

e) Si )(mod nba )(mod nbkak para cualquier entero k

f) Si )(mod nba )(mod nba kk para cualquier entero positivo k .

Page 27: TEORÍA DE NÚMEROS1 Principios. Problemas sin teoría. La "Teoría de números" o "aritmética" estudia las propiedades de los números enteros. Los conceptos teóricos de esta rama

Qué funciona y qué no funciona con congruencias.

Las propiedades anteriores nos permiten trabajar con congruencias prácticamente igual

a como trabajamos con números, pero no todo lo que hacíamos con números funciona

ahora con congruencias:

a) Cancelación de términos:

No funciona en general la cancelación de términos, el "tachar" de toda la vida.

Por ejemplo: )6(mod1242 , pero )6(mod14

)7(mod21217 , pero )7(mod1216

Aunque existe una Regla de cancelación:

Si 1),( nc , entonces )(mod)(mod nbancbca

b) Principio del producto nulo:

No existe tampoco el principio del producto nulo en general. Por ejemplo:

)12(mod034 , pero )12(mod04 y )12(mod03

Pero sí se verifica cuando el módulo es un número primo:

Si p es primo, )(mod0)(mod0)(mod0 pbopapba

Modificaciones en el módulo.

a) Si 1),( dma , entonces

d

mayaxmayax mod)(mod .

b) Si )(mod nba y nd | )(mod dba

c) )(mod nba y )(mod mba mnba ,mod .

d) Si 1,,..., 21 knnn , )...(mod,...,1)(mod 21 ki nnnbakinba

Demostración.

a) yxd

a

d

myxamayaxmmayax |||)(mod

b) )(mod)-(|)-(|)(mod nbabanbamnmnba

c) ),mod)(|,)(|)(mod

)(|)(modmnbabamn

bammba

bannba

d) Es un caso particular de c.

Page 28: TEORÍA DE NÚMEROS1 Principios. Problemas sin teoría. La "Teoría de números" o "aritmética" estudia las propiedades de los números enteros. Los conceptos teóricos de esta rama

Trabajar con congruencias es una técnica muy potente para resolver problemas de Teoría de

Números, como se puede ver en los siguientes ejemplos:

Ejemplo 1.

Demostrar que 41 divide 1220

Solución.

En primer lugar vemos que )41(mod93225

Luego )41(mod8181)41(mod99)41(mod9222244520

Pero, por otro lado, )41(mod1)41(mod81

Luego )41(mod1)41(mod)1)(1()41(mod8181

Finalmente, 12|41)41(mod0)41(mod1112 2020 , tal y como queríamos

ver.

Ejemplo 2.

Determina el residuo al dividir !100!99...!4!3!2!1 entre 12.

Solución.

Observamos que )12(mod024!4 , luego, para todo 4k ,

)12(mod0...650...65!4! kkk

luego

)12(mod!4!3!2!100...0!4!3!2!1!100!99...!4!3!2!1

Y nuestro problema se reduce a encontrar el residuo al dividir 33!4!3!2!1 entre 12,

que es 9.

Un método efectivo para calcular a

b (mod n): “Método de las potencias de dos”

Calculamos las potencias )(mod2 nam

sucesivamente y en orden ascendiente. Después

descomponemos nuestra potencia )(modnab como producto de las anteriores. Como

tantas cosas en Teoría de Números y en general en matemáticas, lo mejor es observar un

ejemplo práctico:

Ejemplo.

Calcular )191(mod3172 .

Solución.

)191(mod932 )191(mod8133224

)191(mod67656133248 )191(mod96448933

2816

)191(mod4892163321632 )191(mod12230433

23264

)191(mod14433264128

Y ahora, puesto que 4832128172 ,

)191(mod170)191(mod816748144)191(mod3333)191(mod3 4832128172

Page 29: TEORÍA DE NÚMEROS1 Principios. Problemas sin teoría. La "Teoría de números" o "aritmética" estudia las propiedades de los números enteros. Los conceptos teóricos de esta rama

Podemos encontrar otros ejemplos de aplicación de este método en las soluciones de los

problemas #6.13 , #11.1 y #6.15.

6.1 MF

Si !n denota el producto de todos los números del 1 al n, ¿Cuál es el residuo de

!...!3!2!1 n

al dividirlo entre 9?

6.2 MF

Encuentra un ejemplo que demuestre que )(mod22 nba no implica )(mod nba .

6.3 F

Determina los residuos cuando 502 y 6541 son divididos entre 7.

6.4 F

Utilizando la teoría de congruencias, demuestra que 12|89 44 y 12|79 48 .

6.5 F

Determina el último dígito de 8019021003 379 .

6.6 MF

Demuestra que para todo INn , el número 122 1211 nn

na es divisible entre 133.

6.7 MF

La cifra de las unidades de 7532137 es:

(A) 1 (B) 3 (C) 5 (D) 7 (E) 9

AHSME 1961 #28

6.8 F

El dígito de las unidades de 100310021001 1373 es:

(A) 1 (B) 3 (C) 5 (D) 7 (E) 9

AHSME 1983 #14

6.9 F

Demuestra que )7(mod1186 k para .3,2,1k

Page 30: TEORÍA DE NÚMEROS1 Principios. Problemas sin teoría. La "Teoría de números" o "aritmética" estudia las propiedades de los números enteros. Los conceptos teóricos de esta rama

6.10 MF

Demuestra que si n es impar, entonces )8(mod12 n

6.11 F

Determina el número de enteros n , 251 n , tales que 232 nn es divisible entre

6.

6.12 F

Demuestra que nn 962 siempre es divisible entre 7, para todo entero positivo n.

6.13 M

Sea nn

na 86 . Determina el residuo cuando 83a se divide entre 49.

AIME 6.13 #6

6.14 F

Determina el residuo al dividir nueves999

9...99...999999 entre 1000.

AIME I 2010 #2

6.15 F

Consideremos el esquema triangular de números ...3,2,1,0 a lo largo de los lados y con

números interiores obtenidos sumando los dos números superiores de la fila anterior.

Las filas 1 a 6 se muestran en el siguiente esquema:

Sea )(nf la suma de los números de la fila n. ¿Cuál es el residuo cuando dividimos

)100(f entre 100?

AHSME 1995 #27

6.16 M

Sea 20082 22008 k . ¿Cuál es el dígito de las unidades de kk 22 ?

(A) 0 (B) 2 (C) 4 (D) 6 (E) 8

AMC12A 2008 #15, AMC 10A 2008 #24

Page 31: TEORÍA DE NÚMEROS1 Principios. Problemas sin teoría. La "Teoría de números" o "aritmética" estudia las propiedades de los números enteros. Los conceptos teóricos de esta rama

6.17 F

Determina el menor entero positivo n tal que n y n107 tienen las dos últimas cifras

iguales.

HMMT 2008 #2

6.18 M

Demuestra que si zyx ,, son enteros cumpliendo 222 3zyx , entonces 0 zyx .

6.19 F

Determina los enteros n tales que 2n divide 218n .

PUMaC 2007/NT #B2

Congruencias con potencias y polinomios.

Congruencias con potencias.

Las propiedades estudiadas en el Tema 4 tienen unas aplicaciones muy importantes en

el estudio de las congruencias:

Dados dos enteros positivos kd , , con kd | , entonces:

a) )(mod)(mod nbanba kk

b) )(mod)(mod nbanba kkdd

c) )(mod nba dd y dk / es impar, )(mod nba kk

Demostración.

a) Basta aplicar kk baba | para todo k.

b) Basta aplicar kkdd babakd || .

c) (*)|)(mod0)(mod dddddd bannbanba

Pero si dk / es impar tenemos kkdd baba | , luego

)(mod)(mod0|(*) nbanbaban kkkkkk

Ejemplo.

Aprovechando que 12552641 744 , demostrar que 12|641 32 .

12|641

)641(mod012)641(mod21)641(mod21

)641(mod21641)641(mod225

)641(mod225)641(mod2225

)641(mod25)641(mod05252641

32

3232324

3243247

32284284284

444444

Page 32: TEORÍA DE NÚMEROS1 Principios. Problemas sin teoría. La "Teoría de números" o "aritmética" estudia las propiedades de los números enteros. Los conceptos teóricos de esta rama

Teorema.

Dado un polinomio con coeficientes enteros 011 ...)( cxcxcxcxp m

m

m ,

entonces:

)(mod)()()(mod nbpapnba .

Demostración.

)(mod)()(

)(mod)(mod)(mod

00

nbcbpacap

nbcacnbanba

m

k

k

k

m

k

k

k

k

k

k

k

kk

6.20 F

Dado cualquier número positivo n, y sea S la suma de sus cifras, demuestra:

a) Sn es divisible entre 9.

b) n es divisible entre 9 si y solo si S es divisible entre 9.

En este problema justificamos el “criterio de divisibilidad del nueve”: Un número es

divisible entre nueve si y solo si las suma de sus cifras es divisible entre 9.

6.21 F

Demuestra el “criterio de divisibilidad del once”: Un número es divisible entre 11 si y

solo si la suma alternada de sus cifras es múltiplo de 11.

Proposición.

Dado un polinomio con coeficientes enteros 011 ...)( cxcxcxcxp m

m

m , diremos

que a es una solución de la congruencia )(mod0)( nxp si )(mod0)( nap .

Si a es una solución de la congruencia )(mod0)( nxp y )(mod nab entonces b

también es una solución de la congruencia )(mod0)( nxp .

Demostración.

)(mod)(0)(mod)()(0)(mod nbpnbpapnba

Page 33: TEORÍA DE NÚMEROS1 Principios. Problemas sin teoría. La "Teoría de números" o "aritmética" estudia las propiedades de los números enteros. Los conceptos teóricos de esta rama

7 Introducción a las ecuaciones diofánticas.

Definición. Ecuación diofántica.

Una ecuación diofántica es aquella en la que únicamente son aceptables soluciones

enteras. En general, no existen métodos "mecánicos" para resolver ecuaciones

diofánticas y precisamente esto las hace muy interesantes para el planteamiento de

problemas.

Antes de empezar a trabajar con los métodos para resolver ecuaciones diofánticas

lineales, vamos a presentar una lista de problemas de ecuaciones diofánticas sin

armadura teórica previa, con solo aplicar el sentido común y los conceptos básicos

del álgebra y la divisibilidad. De esta manera, cogeremos práctica y nos

familiarizaremos con este tipo de problemas.

7.1 M

El número de pares de enteros ),( yx con yx 0 tales que yx 1984 es:

(A) 0 (B) 1 (C) 3 (D) 4 (E) 7

ASHME 1984 #28

7.2 M

El número n se escribe en base 14 como cba , se escribe en base 15 como bca y se

escribe en base 6 como caca , con 0a . Determina el número n en base 10.

AIME I 2018 #2

7.3 F

Determina justificadamente todos los pares de números enteros ),( yx que verifican la

ecuación 200942 yx .

OME Fase Nacional 2009 #4

7.4 F

El número de triples cba ,, de números enteros positivos que satisfacen el sistema

23

44

bcac

bcab

es

(A) 0 (B) 1 (C) 2 (D) 3 (E) 4

AHSME 1984 #20

Page 34: TEORÍA DE NÚMEROS1 Principios. Problemas sin teoría. La "Teoría de números" o "aritmética" estudia las propiedades de los números enteros. Los conceptos teóricos de esta rama

7.5 F

Determina el número de 7-tuplas de números positivos ),,,,,,( gfedcba que satisfacen

el siguiente sistema de ecuaciones:

72

71

70

efg

cde

abc

AIME II 2019 #3

7.6 M

Determina 223 yx si yx, son enteros que satisfacen 517303 2222 xyxy .

AIME 1987 #5

7.7 M

Resuelve la siguiente ecuación diofántica 124

nm

ASHME 1993 #19

7.8 F

Encuentra todos los primos p y q que satisfacen la ecuación 3)( qpqp

Rusia, 2001

7.9 M

Resuelve el siguiente sistema de ecuaciones, con cba ,, enteros.

97

19

cab

cba

ASHME 1997 #28

7.10 F

Resuelve la ecuación con solución entera 1122

xxx

ASHME 1985 #21

7.11 M

Determina todas las tercias de enteros positivos cba ,, tales que 1 cbaabc .

México, 2010

7.12 D

Determina todas las soluciones enteras de la ecuación yx 312

Page 35: TEORÍA DE NÚMEROS1 Principios. Problemas sin teoría. La "Teoría de números" o "aritmética" estudia las propiedades de los números enteros. Los conceptos teóricos de esta rama

7.13 F

Describir todas las soluciones enteras positivas nm, de la ecuación 278 nm

y dar el primer valor de m (si existe) mayor que 1959.

OME 2017 Fase Local #1

Page 36: TEORÍA DE NÚMEROS1 Principios. Problemas sin teoría. La "Teoría de números" o "aritmética" estudia las propiedades de los números enteros. Los conceptos teóricos de esta rama

8 Ecuaciones diofánticas lineales.

Ecuaciones diofánticas lineales.

Son las que tienen la forma cybxa , con cba ,, enteros, y ba, no ambos cero.

Por ejemplo, la ecuación 1863 yx tiene infinitas soluciones:

18)2(61032,10

1866)6(36,6

1816431,4

yx

yx

yx

En general, cualquier valor 183186)3(6)2(33,2 kkkkkykx

Sin embargo, la ecuación 17102 yx no tiene solución: Para cualquier yx, , la parte

izquierda de la ecuación será par, mientras que la parte derecha es un impar.

Teorema fundamental de las ecuaciones diofánticas lineales.

Una ecuación diofántica lineal cybxa tendrá solución si y solo si c|),( ba .

Si 00 , yx es una solución particular de esta ecuación, entonces todas las soluciones

son de la forma

kd

bxx

0 , k

d

ayy

0 para cualquier k entero.

Ejemplo.

Resuelve la ecuación diofántica 100020172 yx

Solución.

4)20,172( d , y 1000|4 , luego esta ecuación tiene solución.

Probando números vemos que 10007205172 , luego 7,5 00 yx es una

solución particular de la ecuación.

Por el Teorema anterior, las soluciones de esta ecuación son todas las parejas de la

forma

)1(5554

205 kkkx

, kky 437

4

1727

, con Zk

o equivalentemente, tomando kq 1 ,

qx 5 , qqqy 435043437)1(437 , con Zq .

8.1 F

Resuelve la ecuación diofántica 397 yx

8.2 F

Un cliente compra una docena de piezas de fruta, manzanas y naranjas, por 1.32€. Si

una manzana cuesta 3 céntimos más que una naranja, y se compraron más manzanas

que naranjas, cuantas piezas de cada fueron compradas?

Page 37: TEORÍA DE NÚMEROS1 Principios. Problemas sin teoría. La "Teoría de números" o "aritmética" estudia las propiedades de los números enteros. Los conceptos teóricos de esta rama

Resolución de ecuaciones diofánticas lineales mediante el Algoritmo de Euclides.

El Algoritmo de Euclides para el cálculo del máximo común divisor de dos números

mediante sucesivas divisiones nos permite resolver ecuaciones diofánticas lineales. Lo

veremos con varios ejemplos:

Ejemplo 1.

Resuelve la ecuación diofántica 5324912173 yx

Solución.

Aplicamos el algoritmo de Euclides:

53)2491,2173(

0535265

532651318

26531862173

318217312491

, y claramente 53|53 , luego existe

solución.

Deshaciendo los pasos del algoritmo de Euclides:

2173824917

21736249162173217312491

21731249162173217312491

318621731217312491265131853

217312491318

31862173265

265131853

Luego 7,8 yx es una solución de la ecuación diofántica del enunciado.

El conjunto de soluciones de la ecuación serán las parejas de la forma:

kkykkx 41753

21737,478

53

24918

8.3 F

Resuelve la ecuación diofántica 33253858 yx mediante el algoritmo de Euclides.

8.4 F

Resuelve la ecuación diofántica 369147258 yx

8.5 F

Resuelve la ecuación diofántica 93360 yx

Page 38: TEORÍA DE NÚMEROS1 Principios. Problemas sin teoría. La "Teoría de números" o "aritmética" estudia las propiedades de los números enteros. Los conceptos teóricos de esta rama

8.6 F

Las medidas (en grados) de los ángulos interiores de un hexágono convexo forman una

sucesión aritmética de enteros positivos. Sea ºm la medida del mayor de los ángulos

interiores de este hexágono. El mayor valor posible de ºm es

(A) 165º (B) 167º (C) 170º (D) 175º (E) 179º

AHSME 1991 #12

8.7 F

Determina un número que, cuando se divide entre 10, deja un residuo de 9, cuando se

divide entre 9 deja un residuo de 8, entre 8 el residuo es 7, y así sucesivamente, hasta

que, finalmente, cuando se divide entre 2, deja un residuo de 1.

ASHME 1951 #37

Page 39: TEORÍA DE NÚMEROS1 Principios. Problemas sin teoría. La "Teoría de números" o "aritmética" estudia las propiedades de los números enteros. Los conceptos teóricos de esta rama

9 Congruencias lineales y sistemas de congruencias lineales.

Congruencia lineal.

Llamamos congruencia lineal a toda ecuación de la forma )(mod nbxa

Diremos que el entero 0x satisface la congruencia lineal )(mod nbxa cuando

)(mod0 nbxa

O equivalentemente: bynxabnyxanbxa 00000 )(mod

Es decir, buscamos soluciones 00 , yx de la ecuación lineal diofántica

bynxa 00

Teorema.

La congruencia lineal )(mod nbxa tiene solución si y solo si bd | , donde

),( nad , en cuyo caso existen d soluciones diferentes (aquí se entiende diferentes

como mutuamente incongruentes), todas ellas de la forma

)1(,...,3,2,, 01030201 dd

nxx

d

nxx

d

nxx

d

nxxx do

Donde 0x es una solución particular de la ecuación.

Ejemplo.

Resuelve la congruencia )42(mod3018 x

Solución.

6)42,18( d , y 30|6 , luego la ecuación anterior tiene seis soluciones.

Por tanteo, vemos que 3014272418 4 es una solución de la ecuación.

Luego las soluciones serán:

40 x

11741)6/42(41 x , efectivamente: 304241981118

181442)6/46(42 x , efectivamente: 304273241818

252143)6/46(43 x , efectivamente: 3042104502518

322844)6/46(44 x , efectivamente: 3042135763218

393545)6/46(45 x , efectivamente: 3042166303918

Las soluciones son: 39,32,25,18,11,4

9.1 F

Resuelve la congruencia )30(mod219 x

9.2 F

Resuelve la congruencia lineal )10(mod73 x

Page 40: TEORÍA DE NÚMEROS1 Principios. Problemas sin teoría. La "Teoría de números" o "aritmética" estudia las propiedades de los números enteros. Los conceptos teóricos de esta rama

Inversos modulares.

Diremos que a y b son inversos módulo n si )(mod1 nba , o equivalentemente,

diremos que b es el inverso de a módulo n.

La congruencia lineal )(mod1 nxa tiene solución si y solo si 1|),( na , es decir,

cuando

1),( na , así pues, existirá el inverso multiplicativo de a si y solo si 1),( na , y será

único modulo n.

Ejemplos.

El inverso de 3 módulo 4 es 3 porque )4(mod1933 .

El inverso de 3 módulo 5 es 2 porque )5(mod1623 .

9.3 F

Determina el inverso de 9 módulo 82.

Page 41: TEORÍA DE NÚMEROS1 Principios. Problemas sin teoría. La "Teoría de números" o "aritmética" estudia las propiedades de los números enteros. Los conceptos teóricos de esta rama

Sistemas de congruencias lineales (caso particular).

Queremos resolver ahora un sistema de congruencias lineales:

rrr mbxa

mbxa

mbxa

mod

...

mod

mod

222

111

En donde vamos a suponer que los módulos im son todos coprimos entre ellos.

Evidentemente, el sistema tendrá solución cuando cada ecuación la tenga

individualmente, y por tanto

kb|kd , donde kkk mad , para todo rk 1

Resolución de sistemas de congruencias lineales mediante el método interactivo.

Ejemplo.

Resuelve el sistema

)3(mod2

)5(mod1

)4(mod3

x

x

x

Solución.

(Más adelante, mediante el Teorema chino del residuo, se verá que la solución existe y

es única mod 60, pues )3,4()3,5()5,4( )

)5(mod3)5(mod52)5(mod24)5(mod134)5(mod1

34)4(mod3

aax

axx

112032543425

)5(mod2)5(mod1216)5(mod3444)5(mod34

bbaxba

aaaa

Luego:

cbb

bbx

3)3(mod0

)3(mod0)3(mod920)3(mod21120)3(mod2

Finalmente:

ccbx 6011113201120 )60(mod . En efecto:

32411 , 12511 , 23311

Page 42: TEORÍA DE NÚMEROS1 Principios. Problemas sin teoría. La "Teoría de números" o "aritmética" estudia las propiedades de los números enteros. Los conceptos teóricos de esta rama

Teorema chino del residuo.

Sean rmmm ,...,, 21 enteros positivos tales que 1, ji mm si ji . Entonces el sistema

de congruencias lineales

rrr mbxa

mbxa

mbxa

mod

...

mod

mod

222

111

Tiene una única solución (módulo el entero rmmm ...21 ).

Y se obtiene siguiendo los siguientes pasos:

Paso 1: Sea rmmmN ...21 ,

Paso 2: Sean rr mNNmNNmNN /,...,/,/ 2211 .

Paso 3: Resolver las congruencias lineales:

111 mod1 myN , 222 mod1 myN , ... , rrr myN mod1

Paso 4: )(mod...222111 NbyNbyNbyNx rrr es la única solución del

sistema.

Ejemplo: El problema de Sun-Tsu.

El Teorema chino del residuo debe su nombre en honor al siguiente problema del siglo I

DC: Determina un número cuyos residuos son 2, 3 y 2 al dividirlo entre 3, 5 y 7,

respectivamente.

Nota: Este mismo problema aparece en las Introductio Arithmeticae del matemático

griego Nicómano de Gerasa, alrededor del 100 DC.

Solución.

Se trata de resolver el sistema de congruencias lineales

)7(mod2

)5(mod3

)3(mod2

x

x

x

Paso 1: 105753 N

Paso 2: 157

105,21

5

105,35

3

105321 NNN

Paso 3: Las congruencias lineales )3(mod135 1 y , )5(mod121 2 y y )7(mod115 3 y

tienen soluciones 21 y , 12 y y 13 y .

Paso 4: 233211531212235 x será solución del sistema módulo 105, y

)105(mod23233

Y por tanto 23 es la única solución del sistema (módulo 105).

En efecto: 27323,35423,23723

Page 43: TEORÍA DE NÚMEROS1 Principios. Problemas sin teoría. La "Teoría de números" o "aritmética" estudia las propiedades de los números enteros. Los conceptos teóricos de esta rama

Ejemplo.

Resolver el sistema de congruencias

)9(mod7

)4(mod2

x

x

Solución.

Claramente 1)9,4( y por tanto el sistema tiene solución.

Paso 1: 3694 N

Paso 2: 94

361 N , 4

9

362 N .

Paso 3: Resolvemos las ecuaciones )4(mod19 1 y y )9(mod14 2 y

Puesto que 14219 , y 1932874 , tenemos que 11 y , 72 y son

soluciones.

Paso 4: La solución es 214774219 x (mod 36) , es decir, 34.

Efectivamente, 28434 , y 73934 ,

Page 44: TEORÍA DE NÚMEROS1 Principios. Problemas sin teoría. La "Teoría de números" o "aritmética" estudia las propiedades de los números enteros. Los conceptos teóricos de esta rama

Sistemas de congruencias lineales (caso general).

Se pueden resolver sistemas de congruencias incluso cuando sus módulos no son

necesariamente coprimos. El criterio es similar al de las ecuaciones diofánticas lineales.

Teorema.

Dado el sistema

)(mod

)(mod

22

11

max

max

Si 21,mm no es divisor de 21 aa , el sistema no tiene solución.

En caso contrario, existe una única solución mod 21,aa

Observamos que el Teorema chino del residuo sería un caso particular de este teorema

cuando 1, 21 mm , pues entonces garantizamos que el sistema tenga solución, y

2121, mmaa .

Ejemplo.

Resolver el sistema

)18(mod11

)12(mod5

x

x

Solución.

Puesto que 6)18,12( , y 5)-(11|6 , existirá una única solución. La vamos a obtener

con el método interactivo.

)18(mod612)18(mod51112)18(mod11512)18(mod11

512)12(mod5

aaax

axx

Esta última congruencia se puede simplificar: 6 divide a 12 y a 6, y además

6)18,6( mcd , luego podemos simplificarla:

)36mod(293652312512

23)3(mod2)3(mod12)18(mod612

kkax

kaaaa

Donde hemos tenido en cuenta que 3618,12

Efectivamente, 521229 , y 1111829

Page 45: TEORÍA DE NÚMEROS1 Principios. Problemas sin teoría. La "Teoría de números" o "aritmética" estudia las propiedades de los números enteros. Los conceptos teóricos de esta rama

Como calcular congruencias cuando el módulo no es primo.

Ejemplo.

Calcular )26(mod20182018 .

Solución.

Puesto que 26 no es primo, no podemos aplicar directamente el PTF. Puesto que

13226 ,

vamos a calcular por separado )2(mod20182018 y )13(mod20182018 , y después

aplicaremos el Teorema Chino del Residuo para determinar el resultado del enunciado.

Está claro que )2(mod02018 y por tanto )2(mod002018 20182018 .

3155132018 , luego 2018|13 , y por tanto podemos aplicar el PTF:

)13(mod1201812 .

Por otro lado, 2168122018 , luego:

)13(mod2018201812018201820182018 221682168122168122018

)13(mod932018)13(mod320183155132018 22

De todo lo anterior tenemos:

)13(mod92018

)2(mod02018

2018

2018

Y aplicamos el Teorema Chino del Residuo:

2

13

26132

2

1

N

N

N

No hace falta resolver la congruencia )2(mod113 1 y pues 01 b .

Resolvemos la congruencia 7)13(mod12 22 yy ,

Luego )26(mod22)26(mod1269720132018 1

2018 y

Observación.

Para calcular potencias elevadas con módulos no primos disponemos de dos técnicas:

El “método de las potencias de dos” y el método que acabamos de ver: descomponer el

módulo y aplicar el Teorema Chino del Residuo. Es importante dominar estas dos

técnicas, pues son la clave para resolver muchísimos problemas de Aritmética. Se

propone resolver el siguiente problema mediante las dos técnicas anteriores:

9.4 F

Determina los dos últimos dígitos de 10321032 .

HMMT 2009

Page 46: TEORÍA DE NÚMEROS1 Principios. Problemas sin teoría. La "Teoría de números" o "aritmética" estudia las propiedades de los números enteros. Los conceptos teóricos de esta rama

Congruencias lineales que se resuelven mediante sistemas de congruencias lineales.

El siguiente resultado nos puede ser útil para resolver congruencias lineales:

)(mod)(mod)(mod mbxaynbxamnbxa

En efecto: )(mod)(mod nbxabnkmbnmkxamnbxa

Existe un recíproco: mnbxambxaynbxa ,mod)(mod)(mod

Ejemplo.

Resolver la congruencia lineal )276(mod917 x .

Solución.

Puesto que 2334276 , la ecuación anterior es equivalente a resolver el sistema de

congruencias

)23(mod917

)4(mod917

)3(mod917

x

x

x

o equivalentemente:

)23(mod917

)4(mod1

)3(mod0

x

x

x

)3(mod0x equivale a decir que ax 3 , luego sustituyendo en la segunda ecuación y

multiplicando por 3 ambos lados:

912334)4(mod3)4(mod39)4(mod13 baxbaaaa

Sustituyendo en la tercera ecuación:

)276(mod33332769242769)223(12

223)23(mod2)23(mod63

)23(mod63)23(mod1720)23(mod144204

)23(mod9153204)23(mod991217)23(mod917

xkkkx

kbbb

bbb

bbx

Efectivamente, 927625613317

Page 47: TEORÍA DE NÚMEROS1 Principios. Problemas sin teoría. La "Teoría de números" o "aritmética" estudia las propiedades de los números enteros. Los conceptos teóricos de esta rama

Congruencias no lineales.

Ejemplo.

Resuelve la ecuación )144(mod12 x

Solución.

Puesto que 916144 , y 1)9,16( , podemos descomponer la ecuación anterior en el

sistema no lineal

)9(mod1

)16(mod1

2

2

x

x

)16(mod12 x tiene 4 soluciones: 1x o 7 (mod 16)

)9(mod12 x tiene 2 soluciones: 1x (mod 9)

Luego tenemos ocho alternativas:

i) )16(mod1x y )9(mod1x

ii) )16(mod1x y )9(mod1x

iii) )16(mod1x y )9(mod1x

iv) )16(mod1x y )9(mod1x

v) )16(mod7x y )9(mod1x

vi) )16(mod7x y )9(mod1x

vii) )16(mod7x y )9(mod1x

viii) )16(mod7x y )9(mod1x

Podemos ir resolviendo cada caso mediante el Teorema chino del residuo.

Independientemente del caso, 1)9,16( , luego todos los ocho sistemas tienen solución.

Además: 16,9 21 NN , 9)16(mod19 11 yy , 4)9(mod116 22 yy

i) )144(mod1)144(mod1451416199 x

ii) )144(mod17)1(416199 x

iii) )144(mod171416)1(99 x

iv) )144(mod1)144(mod145)1(416)1(99 x

v) )144(mod55)144(mod6311416799 x

vi) )144(mod71)144(mod503)1(416799 x

vii) )144(mod71)144(mod73)144(mod5031416)7(99 x

viii) )144(mod55)144(mod631)1(416)7(99 x

9.5 F

Calcula los tres últimos dígitos de 20061211 2005...20052005

Senior Hanoi Open MO 2006

Page 48: TEORÍA DE NÚMEROS1 Principios. Problemas sin teoría. La "Teoría de números" o "aritmética" estudia las propiedades de los números enteros. Los conceptos teóricos de esta rama

10 El pequeño Teorema de Fermat.

Teorema. Pequeño Teorema de Fermat (PTF).

Si p es primo,

a) )(mod paa p para cualquier entero a .

b) Si ap | , entonces )(mod11 pa p

Nota: El recíproco no es cierto: )(mod11 nan para cierto entero a n primo. Esto

se estudiará detenidamente en una observación más adelante.

Demostración.

a) Aunque en el próximo tema veremos que este teorema es un caso particular de

aplicación de la función Phi de Euler, vamos a presentar aquí una demostración directa.

Caso 1: Si 0a . aaa p 00 y está claro que entonces )(mod paa p .

Caso 2: Si 0a . Vamos a demostrarlo por inducción en a :

Si 1a , 11 ppa y está claro que entonces )(mod paa p .

Supongamos cierto )(mod paa p , queremos ver que entonces es cierto para 1a .

Aplicando el binomio de Newton:

11

...1

1 1

a

p

pa

paa ppp

Pero

k

pp | para todo p k1 (por la definición de

)!(!

!

kpk

p

k

p

al ser p primo,

estará en el numerador, pero no en el denominador) , y por tanto

)(mod11 paa pp .

Finalmente, aplicando la hipótesis de inducción:

)(mod1)(mod11 papaa pp ,

tal y como queríamos ver.

Caso 3: Si 0a . Si 2p , entonces

)2(mod2|2|2)2(mod)( 222222 aaaaaaaaaaaa

Si 2p , entonces p es impar, y por tanto: )(mod)(mod)()( papaaa pp

En donde hemos aplicado el “Caso 1” pues a es positivo.

b) Aplicando el apartado anterior, 1|)(mod 1 ppp aaaappaa .

Aplicando el Lema de Euclides, puesto que, por hipótesis, ap | , deducimos que

1| 1 pap , o equivalentemente, papa pp mod1mod01 11

Page 49: TEORÍA DE NÚMEROS1 Principios. Problemas sin teoría. La "Teoría de números" o "aritmética" estudia las propiedades de los números enteros. Los conceptos teóricos de esta rama

El PTF se puede aplicar al cálculo de congruencias con potencias de números, como en el

siguiente ejemplo:

Ejemplo 1.

Demostrar que )11(mod4538 .

Solución.

Aplicamos el PTF para garantizar que )11(mod1510 , luego:

(*))11(mod5)11(mod515555 8838310810338

Por otro lado )11(mod32552 , y por tanto:

)11(mod4)11(mod81)11(mod3)11(mod5(*) 442

Donde hemos aplicado que 411781

Ejemplo 2.

Calcular )13(mod7121

Solución.

Puesto que 7|13 podemos aplicar el PTF para garantizar que )13(mod1712 .

Puesto que 11012121 )13(mod7717777 10101211012121

El PTF se puede utilizar también para testear si un número n es primo o no. Si encontramos un

entero a tal que )(modnaan , entonces seguro que n no es primo (pues en caso contrario

contradeciría el PFT). Veamos esto en el siguiente ejemplo:

Ejemplo.

Demostrar que 117 no es primo.

Solución.

Tomamos 2a . Sabemos que )117(mod1112827 , y que )117(mod4121 ,

luego:

)117(mod22412111112 1682888216167

)117(mod2)117(mod222222 2151651675167117

Pero )117(mod44114111211122 33721

Con lo que, finalmente llegamos a )117(mod2442117 , y por tanto 117 debe ser un

número compuesto (de hecho: 913117 ).

Page 50: TEORÍA DE NÚMEROS1 Principios. Problemas sin teoría. La "Teoría de números" o "aritmética" estudia las propiedades de los números enteros. Los conceptos teóricos de esta rama

Proposición. Descomposición de módulos.

Si p y q son primos diferentes tales que )(modqaa p y )(mod paaq , entonces:

)(mod qpaa qp

)(mod paa qpq por el Corolario al PTF, y )(mod paaq por hipótesis, luego:

Luego )(mod)(mod papaaa qpqqp , es decir: aap qp |

Con un razonamiento similar llegamos a aaq qp | , y por tanto:

aaqp qp | , o equivalentemente: )(mod0 qpaa qp

Ejemplo.

Demostrar que )341(mod12340 .

Solución.

Aquí 3111341 , y por otra parte, 133311024210 , luego )31mod(1210 , y por

tanto:

)31(mod212222 1011

Por otro lado, )11(mod121931110242 1010 , y por tanto:

)11(mod21222222 331031031

Y aplicando la proposición anterior: )341(mod222 3111341 , y cancelando un factor 2

llegamos al resultado deseado: )341(mod12340

Observación. Los números pseudoprimos y el recíproco del PTF.

El PTF dice que n primo )(mod22 nn , y acabamos de ver que )341(mod22341

y sin embargo 3111341 no es primo, es decir un contraejemplo para el recíproco del

PTF.

Este dato es curioso porque 341n es el primer compuesto tal que )(mod22 nn , es

decir, sirve como contraejemplo del recíproco del PTF, es decir:

n primo )(mod22 nn para 3402 n

Esta es la razón por la que los matemáticos chinos antiguos creyeron, equivocadamente,

que )(mod22 nn caracterizaba los números primos.

Los números n tales que )(mod22 nn , es decir 22| nn se denominan

pseudoprimos. Hay infinitos pseudoprimos y los más pequeños son 341, 561, 645 y

1105.

Page 51: TEORÍA DE NÚMEROS1 Principios. Problemas sin teoría. La "Teoría de números" o "aritmética" estudia las propiedades de los números enteros. Los conceptos teóricos de esta rama

Aplicación del PTF a la resolución de congruencias no lineales.

Ejemplo 1.

Determina una solución de la congruencia )11(mod4103 x

Solución.

Puesto que, aplicando el PTF, )11(mod110 x

)11(mod3331031010103 xxxxx

Luego hemos reducido nuestro problema a resolver la congruencia )11(mod3 xx

Probando valores 1x , 2x , 3x ,... llegamos a )11(mod453 , y por tanto:

)11(mod5x

Ejemplo 2.

Determina una solución de la congruencia )29(mod686 x .

Solución.

Aplicando el PTF, sabemos que )29(mod128 x para todo x .

Luego )29(mod22328228386 xxxxx , luego hemos reducido nuestro problema a

resolver la congruencia )29(mod62 x .

Vamos probando valores 1x , 2x , 3x ,... hasta llegar a

)29(mod66229648 xx

Luego una solución es )29(mod8x

Nota: Existe otra solución: )29(mod66152944121 2 xx

Para encontrar estas soluciones se puede hacer el siguiente planteamiento:

)29(mod646 , y por tanto, la ecuación )29(mod62 x es equivalente a

)29(mod642 x ,

y ahora:

8

8)29(mod088)29(mod064)29(mod64 22

x

xxxxx

y finalmente: )29(mod218

10.1 F

Aplicando el PTF, determina:

a) )7(mod331 b) )7(mod235 c) )17(mod128129

10.2 F

Dividimos el número 10002 entre 13. ¿Cuál es el residuo?

AHSME 1972 #31

Page 52: TEORÍA DE NÚMEROS1 Principios. Problemas sin teoría. La "Teoría de números" o "aritmética" estudia las propiedades de los números enteros. Los conceptos teóricos de esta rama

10.3 F

Utilizando el PTF, demuestra que 17 divide a 111104

10.4 F

Demuestra que si 1)35,( a , entonces )35(mod112 a

10.5 F

Sea 41 a , 14 na

na , 1n . Determina el residuo cuando 100a se divide entre 7.

10.6 F

Demuestra que, si 1)42,( a , entonces 1|873168 6 a .

10.7 F

Determina )7(mod65432 6050403020

10.8 D

En los años 60, tres matemáticos americanos demostraron que una de las conjeturas de

Euler era falsa al encontrar un entero positivo n tal que

55555 2784110133 n

Determina n.

AIME 1989 #9

Nota: Se presentan dos soluciones, pero ninguna de las dos es completa: Son argumentos que justifican

que un cierto n es el mejor candidato posible.

10.9 F

Determina los números primos p para los cuales 129 p es múltiplo de p

Page 53: TEORÍA DE NÚMEROS1 Principios. Problemas sin teoría. La "Teoría de números" o "aritmética" estudia las propiedades de los números enteros. Los conceptos teóricos de esta rama

11 La función Phi de Euler.

Definición. La función Phi de Euler.

Dado un número natural 1n , )(n indica el número de números naturales menores

que n y coprimos con n.

Por ejemplo, 8)30( porque el número de naturales coprimos con 30 son 1, 7, 11, 13,

17, 19, 23 y 29. De la misma manera vemos que

,4)5(,2)4(,2)3(,1)2(,1)1(

...6)7(,2)6(

Observamos que 1)1( porque 1)1,1( mcd , sin embargo, si 1n , 1),( nnnmcd .

Observamos p es primo si y solo si 1)( np .

Con Mathematica:

Cálculo explícito de la función Phi de Euler.

Si la descomposición en factores primos de n es rk

r

kkpppn ...21

21 , entonces:

r

i i

k

r

k

r

kkkk

pnppppppn rr

1

11

22

1

11

11...)( 2211

Ejemplo 1.

Calcular )360(

Solución.

965

11

3

11

2

11360)360(532360 23

Ejemplo 2.

Calcular )1001(

Solución.

72013

11

11

11

7

111001)1001(131171001

Page 54: TEORÍA DE NÚMEROS1 Principios. Problemas sin teoría. La "Teoría de números" o "aritmética" estudia las propiedades de los números enteros. Los conceptos teóricos de esta rama

Propiedad de la función Phi de Euler.

Fijemos un número n y un divisor suyo: Por ejemplo, 30n y 5d .

Estudiemos el conjunto dnmnmSd ),(, . En la siguiente tabla vemos que

20,55 S

nm 1 )30,(m 5)30,( m 2)6(

1 1

2 2

3 3

4 2

5 5 155 1

6 6

7 1

8 2

9 3

10 10 2510 2

11 1

12 6

13 1

14 2

15 15 3515 3

16 2

17 1

18 6

19 1

20 10 4520 4

21 3

22 2

23 1

24 6

25 5 555 5

26 2

27 3

28 2

29 1

30 30 6530 6

Consideremos ahora 6/ dn y )6( . El conjunto de números menores que 6 y

coprimos con 6 es 5,1 , luego 2)6( .

Vemos que existe una biyección perfecta entre 5S y los coprimos de 6, y por tanto

)/( dnSd

Por otro lado, todo número nm pertenece a algún dS con nd | , y por tanto:

n|n|

)/(dd

d dnSn

Existe también una biyección nn

dnd || y por tanto llegamos a la siguiente

propiedad:

n|

)(d

dn

Page 55: TEORÍA DE NÚMEROS1 Principios. Problemas sin teoría. La "Teoría de números" o "aritmética" estudia las propiedades de los números enteros. Los conceptos teóricos de esta rama

Teorema de Euler.

)(mod1)( na n si 1),( na

Nota histórica.

La primera demostración del PTF , )(mod11 pa p si ap | , fue dada por Euler en

1736. El propio Euler presentó en 1760 este teorema, que es una generalización del PTF

porque si n es primo entonces 1)( pn , y appa |1),( .

Observación.

La condición 1),( na es necesaria, pues si 1),( na , ya vimos en el capítulo 8 que la

congruencia )(mod1 nxa no tiene solución, y por tanto no puede existir ningún k tal

que )(mod1 nak , pues en ese caso 1 kax sería solución de la congruencia

)(mod1 nxa .

Ejemplo.

Tomando 30n y 11a , 8)30( y )30(mod1118

En efecto, )30(mod111111)30(mod112111 44282

Orden de un entero.

El Teorema de Euler indica que si 1),( na , la secuencia

,...,,, 432 aaaa

siempre alcanza el 1 (y por lo tanto se vuelve periódica), y lo alcanza en )(na .

Naturalmente, )(n no es necesariamente el primer número k para el cual 1ka . El

menor exponente k para el cual 1ka se denomina orden de a (módulo n) y se

estudiará en el apartado siguiente.

El Teorema de Euler y el PTF nos permiten reducir potencias muy grandes módulo n.

Ejemplo.

Determina los dos últimos dígitos de la expresión decimal de 2563 .

Solución.

Está claro que este problema implica estudiar )100(mod3256 , y aquí nos puede ayudar

el Teorema de Euler:

405

11

2

1110052)100( 22

, y puesto que 1)100,3( , podemos aplicar el

Teorema de Euler: )100(mod1340

Luego )100(mod3)100(mod33)100(mod33 161664016640256 .

)100mod(2137216161333)100mod(6165613 88168

Por lo tanto, el número 2563 acaba en "21".

Page 56: TEORÍA DE NÚMEROS1 Principios. Problemas sin teoría. La "Teoría de números" o "aritmética" estudia las propiedades de los números enteros. Los conceptos teóricos de esta rama

11.1 F

¿Para cuántos enteros i , cumpliendo 10001 i , existe un entero j , cumpliendo

10001 j , tal que i es un divisor de 12 j ?

11.2 M

Determina los ocho últimos dígitos de la expansión binaria de 198627 .

11.3 D

Determina los tres últimos dígitos de

12...200620072008

Nota:

12...200620072008 se define recursivamente: 200721 2008,...,3,2,1 2008321

aaaaaaa

PuMAC (Princeton University Mathematics Competition)

11.4 D

Definimos xxxxxf )( . Determina los dos últimos dígitos de

)20()19()18()17( ffff

PuMAC 2008

Nota: )(xf se define recursivamente: )(

4

)(

2131 )()(,...,)(,)(

xfxfxxfxfxxfxxf .

Indicación: Si 1),( na , entonces )(mod)(mod naa nbb y podemos reducir el cálculo

a determinar )(mod nb .

Page 57: TEORÍA DE NÚMEROS1 Principios. Problemas sin teoría. La "Teoría de números" o "aritmética" estudia las propiedades de los números enteros. Los conceptos teóricos de esta rama

12 Fraccions algebraiques i equacions racionals.

Una fracció algebraica és una divisió de dos polinomis: )(

)(

xq

xp

12.1 El domini de definició d'una fracció algebraica.

Una fracció algebraica )(

)(

xq

xp està definida sempre que 0)( xq

o, dit d'una altra manera, no estarà definida quan 0)( xq

Exemple resolt.

Determina el domini de definició de 2

12 xx

Solució:

Aquesta expressió no estarà definida quan 022 xx , és a dir, quan 2x o 1x

12.1.1 Determina el domini de definició de les següents expressions:

a) 2

12 x

b) 1

12 xx

c) 32

12 xx

Page 58: TEORÍA DE NÚMEROS1 Principios. Problemas sin teoría. La "Teoría de números" o "aritmética" estudia las propiedades de los números enteros. Los conceptos teóricos de esta rama

12.2 Simplificació de fraccions algebraiques.

Exercici resolt.

Simplifica a) 86

42

2

xx

x b)

ba

b

126

26

Solució:

a)Factoritzem numerador i denominador:

)4)(2(

)2)(2(

86

42

2

xx

xx

xx

x

"Tatxem" el factor repetit a dalt i a baix, afegint la condició 2x al domini de definició

de l'expressió:

4

2

)4)(2(

)2)(2(

x

x

xx

xx si 2x

b) 6

1

)2(6

2

126

2

ba

ba

ba

ba si baba 202

Exercici resolt.

Simplifica xxx

xax

12102

)36(1023

23

Solució:

1

)6(5

)1)(6(2

)6)(6(25

12102

)36(10 23

23

23

x

xax

xxx

xxax

xxx

xax si 1,0 x

12.2.1

Simplifica les següents fraccions algebraiques:

a) y

y

35

30 b)

65

92

2

xx

x c)

bx

bx

84

2

d) 53

753

3

18

cba

cba e)

3212

452

2

xx

xx f)

xy

yx

2

2

g) )9)(1(2

)5)(1(33

4

aaa

aaa h)

10

)10( 3

x

x

12.2.2 Simplifica les següents fraccions algebraiques:

a) 34

122

2

xx

xx b)

152

1032

2

xx

xx c)

76

21102

2

xx

xx

d) xx

xx

6

24102

2

e)

352

142

2

bb

b f)

22

22

44

92820

yxyx

yxyx

Page 59: TEORÍA DE NÚMEROS1 Principios. Problemas sin teoría. La "Teoría de números" o "aritmética" estudia las propiedades de los números enteros. Los conceptos teóricos de esta rama

12.2.3

Simplifica les següents fraccions algebraiques:

a) 2

253 2

x

xx b)

1

32 2

x

xx c)

3

34 22

x

xx

d) 13

51682

23

xx

xxx e)

3

12345

56

x

xxx f)

54

10362

23

xx

xxx

12.2.4 Simplifica les següents fraccions algebraiques:

a) 32

32

bb

b b)

a

a

14

7 3

c) 54

232

2

xx

xx d)

x

yx

30

6 2

e) yx

zyx6

249 f)

32

3

60

20

yzx

xyz g)

aa

a

3

82

h) 63

12

x

x

i) 2

2

2

510

x

xx j)

5

252

x

x i)

84

42

b

b l)

1

332

x

x

m) 149

72

aa

a n)

246

862

x

xx o)

65

672

2

yy

yy

Page 60: TEORÍA DE NÚMEROS1 Principios. Problemas sin teoría. La "Teoría de números" o "aritmética" estudia las propiedades de los números enteros. Los conceptos teóricos de esta rama

12.3 Suma i resta de fraccions algebraiques.

Si les fraccions tenen comú denominador: Sumem o restem els numeradors i deixem el

denominador comú.

Exercici resolt.

Realitza la següent suma de fraccions algebraiques: 7

14

7

23 22

x

xx

x

xx

Solució:

7

334

7

1423

7

14

7

23 22222

x

xx

x

xxxx

x

xx

x

xx

Molta atenció quan restem polinomis! Recorda que generem un parèntesi, i que

hem de canviar de signe tot el que hi ha a dins.

Per exemple: 332153332153 2222 xxxxxxxx

Exercici resolt.

Realitza la següent resta de fraccions algebraiques: 52

42

52

35

y

y

y

y

Solució:

52

13

52

4235

52

4235

52

42

52

35

y

y

y

yy

y

yy

y

y

y

y

12.3.1 Calcula:

a) 22 2

3

2

5

y

x

y

x b)

yx

yx

yx

yx

32

c) 103

52

103

44 22

x

xx

x

xx d)

xx

xx

xx

xx

32

73

)32(

)1(2

2

e) )3)(2(

48

6

342

xx

x

xx

x f)

aa

a

aa

aa

aa

aa

122

2

122

432

)6(2

452

2

2

22

g) )2)(4(

435

86

32

86

18 2

2

2

2

2

xx

xx

xx

xx

xx

xx

Page 61: TEORÍA DE NÚMEROS1 Principios. Problemas sin teoría. La "Teoría de números" o "aritmética" estudia las propiedades de los números enteros. Los conceptos teóricos de esta rama

Exercici resolt.

Calcula la següent suma de fraccions algebraiques: 3

5

2

3

b

b

b

b

Solució:

Si els denominadors són diferents hem de passar primer a comú denominador, calculant

el mínim comú múltiple (mcm) dels denominadors.

)3)(2()3,2( bbbbmcm

)2)(3(

)2(5

)3)(2(

)3(3

2

2

3

5

3

3

2

3

3

5

2

3

bb

bb

bb

bb

b

b

b

b

b

b

b

b

b

b

b

b

Un cop les hem passat a comú denominador, només cal sumar els numeradors i deixar el

denominador comú:

)3)(2(

8

)3)(2(

10593

)3)(2(

10593

)3)(2(

)2(5)3(3

)2)(3(

)2(5

)3)(2(

)3(3

222

22

bb

bb

bb

bbbb

bb

bbbb

bb

bbbb

bb

bb

bb

bb

Exercici resolt.

Calcula la següent suma de fraccions algebraiques: 32

13

127

522

xx

x

xx

x

Solució:

)1)(3)(4()1)(3(32

)3)(4(127

2

2

xxxmcm

xxxx

xxxx

Passem a comú denominador:

)4)(1)(3(

)4)(13(

)1)(3)(4(

)1)(5(

4

4

)1)(3(

13

1

1

)3)(4(

5

)1)(3(

13

)3)(4(

5

32

13

127

522

xxx

xx

xxx

xx

x

x

xx

x

x

x

xx

x

xx

x

xx

x

xx

x

xx

x

Un cop passades a comú denominador, sumem els numeradors i deixem el denominador

comú:

)1)(3)(4(

974

)1)(3)(4(

)4)(13()1)(5(

)4)(1)(3(

)4)(13(

)1)(3)(4(

)1)(5(

2

xxx

xx

xxx

xxxx

xxx

xx

xxx

xx

Page 62: TEORÍA DE NÚMEROS1 Principios. Problemas sin teoría. La "Teoría de números" o "aritmética" estudia las propiedades de los números enteros. Los conceptos teóricos de esta rama

Exercici resolt.

Calcula: 7

5

7

3

x

x

x

x

Solució:

De vegades els denominadors semblen diferents, però són pràcticament iguals. Per

exemple, x7 i 7x són el mateix polinomi canviat de signe.

x

x

x

xx

x

x

x

x

x

x

x

x

x

x

x

x

7

2

7

53

7

5

7

3

)7(

5

7

3

7

5

7

3

12.3.2 Realitza les següents operacions:

a) 2

3

1

5

b

b

b

b b)

3

2

2

7

a

a

a

a c)

3

5

3

14

x

x

x

x

d) 4

1332

y

y

y

y e)

6

2

)6)(1(

2

y

y

yy

y f)

3

2

)3)(3(

12

a

a

aa

a g)

86

2

23

722

aa

a

aa

a h)

44

2

96

622

bbbb

i) 33

2

4

x

x

x

x j)

4

7

4

5

x

x

x

x

12.3.3 Calcula:

a) 2

3

23

22

xxx

x b)

12

4

209

322

bb

b

bb

b

c) 1

3

32

42

aaa

a d)

166

3

124

122

yy

y

yy

y

e) 3127

3 2

2

y

y

yy

y f)

4

5

149

322

x

x

xx

x

12.3.3 Calcula:

a) 23

2

65

3

34

1222

xx

x

xx

x

xx

x

b) 12

1

82

3

86

4222

xx

x

xxxx

x

c) 45

3

43

3

1

2222

yy

y

yy

y

y

y d)

2345 248

2

42

3

xxxx

e) 6

2

124

2

189

2222

aa

a

aa

a

aa

a f)

65

4

6

222

yy

y

yy

y

g) xx

x

x

x

124

1

9

222

Page 63: TEORÍA DE NÚMEROS1 Principios. Problemas sin teoría. La "Teoría de números" o "aritmética" estudia las propiedades de los números enteros. Los conceptos teóricos de esta rama

Exercici resolt.

Calcula: 1

73

x

Solució:

1

7

1

3

1

73

xx

1)1,1( xxmcm

1

43

1

733

1

7)1(3

1

7

1

)1(3

1

1

1

7

1

1

1

3

1

7

1

3

x

x

x

x

x

x

xx

x

xx

x

x

12.3.4 Calcula:

a) 6

343

2

y

yyy b)

6

38

x c)

2

46

y

y

d) 7

41

x e)

6

53

x f)

1

42

2

x

xx

g) 3

5243

2

y

yyy h)

1

42

2

x

xx i)

2

526

b

bb

Si les fraccions són senzilles, podem "tirar de fórmula" per sumar fraccions sense haver

de determinar el mínim comú múltiple dels denominadors:

db

cbda

d

c

b

a

12.3.5 Calcula les següents sumes i restes, simplificant els resultats obtinguts:

a) mm 3

12

3

9 b)

21

12

21

5 xx

c)

xx

93

d) 44

2 tt

e)

2

64

2

3

yy f)

8

14

8

2 xx

g) 14

1

14

3

c

c

c

c h)

2

6

2

7

k

k

k

k i)

5

3

5

2

x

x

x

j) n

n

n

n

23

6

32

3

k)

2

4

2

23

dd l)

4

8

4

a

a

a

a

m) 55

1

55

2

n

n

n

n n)

1

52

1

4

x

x

x

x o)

2

122

2

9

x

x

x

x

Page 64: TEORÍA DE NÚMEROS1 Principios. Problemas sin teoría. La "Teoría de números" o "aritmética" estudia las propiedades de los números enteros. Los conceptos teóricos de esta rama

Exercici resolt.

Calcula 23

4

2

5

yy

Solució: 22 6)3,2( yyymcm

22222 6

815

6

8

6

15

)2(3

)2(4

)3(2

)3(5

3

4

2

5

y

y

yy

y

yyy

y

yy

Exercici resolt.

Calcula 2

5

1

xx

x

Solució:

)2(1

57

)2(1

552

)2(1

55

)2(1

2

)2(1

15

)2(1

)2(

2

5

1

)2)(1()2,1(

22

2

xx

xx

xx

xxx

xx

x

xx

xx

xx

x

xx

xx

xx

x

xxxxmcm

12.3.6 Calcula les següents sumes i restes, simplificant els resultats obtinguts:

a) xx 10

2

2

1 b)

xx

2

7

1 c)

22

510

yxy d)

aa

793

e) 2

5

63

2

xx f)

4

2

82

7

xx g)

441

2

x

x

x

x

12.3.7 Calcula les següents sumes i restes, simplificant els resultats obtinguts:

a) 3

3

6

5

xx b)

3

5

3

4

x

x

x c)

4

2

16

72

xx

x

d) 100

3

10 2

xx

x e)

651

42

xx

x

x

x

Page 65: TEORÍA DE NÚMEROS1 Principios. Problemas sin teoría. La "Teoría de números" o "aritmética" estudia las propiedades de los números enteros. Los conceptos teóricos de esta rama

12.4 Multiplicació i divisió de fraccions algebraiques.

La multiplicació de fraccions algebraiques es fa directament: "Numerador per numerador i

denominador per denominador".

La divisió de fraccions algebraiques es fa directament, "multiplicant en creu".

Exemple resolt:

22)1(24

1

1

2

4

1

)1)(23(

)23(2

4

1

23

)23(2

2

2

2

2

x

x

x

x

xx

x

xxx

xx

xxx

xx

12.4.1 Realitza les següents divisions:

a) 5

:1

2 2

x

xx

x

x b)

1:

1

12

2

x

x

x

xx

Page 66: TEORÍA DE NÚMEROS1 Principios. Problemas sin teoría. La "Teoría de números" o "aritmética" estudia las propiedades de los números enteros. Los conceptos teóricos de esta rama

12.5 Equacions amb fraccions algebraiques sense sumes.

Exercici resolt.

Resol l'equació: 5

2

1

3

xx

Solució:

Aquesta equació té com a condició implícita que 1x i 5x perquè amb aquests

dos valors s'anul·laria algun denominador.

I sempre amb aquesta condició puc multiplicar en creu:

131522322153)1(2)5(35

2

1

3

xxxxxxx

xx

Comprovem la solució obtinguda: 4

1

4

1

8

2

12

3

513

2

113

3

La solució és 13x .

Exercici resolt.

Resol l'equació 3

104

3

73

a

a

a

a

Solució:

Si no som prou disciplinats: "tatxem" els denominadors perquè són iguals.

3371043104733

104

3

73

aaaaaa

a

a

a

a

i la solució és 3a . Però hem de tenir en compte que aquest valor fa anul·lar els

denominadors, i per tant no és vàlid!

0

2

0

2

33

1034

33

733

Aquesta equació no té cap solució.

12.5.1 Resol les següents equacions:

a) 1

1

2

3

a

a

a

a b)

2

3

2

1

y

y

y

y c)

2

1

6

2

x

x

x

x d)

3

4

2

13

m

m

e) 12

4

x

12.5.2

Resol les següents equacions:

a) 2

3

2

422

aaaa

b) 86

3

)2(

22

bbbb

Page 67: TEORÍA DE NÚMEROS1 Principios. Problemas sin teoría. La "Teoría de números" o "aritmética" estudia las propiedades de los números enteros. Los conceptos teóricos de esta rama

12.6 Equacions amb sumes o restes de fraccions algebraiques.

Hem de passar a comú denominador.

Exercici resolt.

Resol l'equació 212

3

x

x

x

Solució:

En aquesta equació estem suposant que 0x i 1x , perquè per a aquests dos valors

s'anul·larien els denominadors.

)1(2)1,2( xxxxmcm

)1(2

)1(22

)1(2

2)1(3

)1(2

)1(22

)1(2

2

)1(2

)1(3

)1(2

)1(22

2

2

11

1

2

32

11

1

2

3

22

xx

xx

xx

xx

xx

xx

xx

x

xx

x

xx

xx

x

x

x

x

x

x

xx

x

x

x

x

Ara podem aprofitar que 0x i 1x per a simplificar els denominadors:

)1(42)1(3)1(2

)1(22

)1(2

2)1(3 22

xxxx

xx

xx

xx

xx

i resoldre l'equació resultant, ara sense denominadors:

2/1

30372044233

44233)1(42)1(3

222

222

x

xxxxxxx

xxxxxxxx

Comprovem les solucions:

22

4

2

3

2

1

2

3

6

3

1)3(

3

)3(2

3

2132/1

2/1

1

3

1)2/1(

2/1

)2/1(2

3

Les solucions són 2/1,3 xx

Exercici resolt.

Resol l'equació 31

43

1

a

a

a

a

Solució:

Passem a comú denominador:

143333343

)1(34331

433

1

43

1

aaaaaaa

aaaa

aa

a

a

a

a

Tanmateix, la solució 1a no és una solució vàlida per a la nostra equació perquè fa

anul·lar els denominadors. Per tant, aquesta equació no té cap solució.

Page 68: TEORÍA DE NÚMEROS1 Principios. Problemas sin teoría. La "Teoría de números" o "aritmética" estudia las propiedades de los números enteros. Los conceptos teóricos de esta rama

12.6.1

a) 6

10

23 2

xxx

x

x

x b)

2

48

1

3

2

42

2

aa

aa

a

a

a

a

c) 45

194

1

2

4

12

xx

x

xx d)

23

443

12

42

2

xx

xx

x

x

x

x

12.6.2

a) kkk

1

3

1

6

122 b)

22 2

111

nnn c)

22

1

6

1

6

1

bbb

d) 222 2

4

2

3

4

6

b

b

bb

b

e) 1

5

61

xx f)

vv

v

vv

v

v 5

567

5

123122

g) mmmmm

22

511 h)

8

71

8

1

nn

i) 2

6

107

1

2

12

xxxx

j) 4

427

4

21

v

v

v

v

k) 15

1

5

4

ss

s l)

x

x

x

xx

3

6

3

2451

2

m) x

x

xx

1

2

11

2

n)

1

61

8

5

nn

n o)

ssss

s

2

11

2

522

p) 17

5

12

x

x

xx q)

2

31

3

3

aa

a r)

1

61522

p

p

pppp

p

s) 223

1

5

4

5

5

nnnn

12.6.3

Resol les següents equacions:

a) 3

14

3

2

yy b)

nn

54 c) x

x27

3

d) 6

31

tt e)

xx

x

x 9)7(

2

f)

2

42

x

xx

g) 348

k

k

k

k h)

4

11

a

a

a

a i) 2

1

2

1

3

n

n

n

n

j) 4

1

46

5

w

w

w k)

xx

x 1

2

l)

3

11

n

n

n

n

m) 4

2

8

x

x

x n)

2

11

2

3

y

y

y

y o)

43

2

4 c

c

c

Page 69: TEORÍA DE NÚMEROS1 Principios. Problemas sin teoría. La "Teoría de números" o "aritmética" estudia las propiedades de los números enteros. Los conceptos teóricos de esta rama

12.6.4 Resol les següents equacions:

a) 11

5

1

aa

a b)

222

1266

vvv

v

c)

22

341

nnn

d) 22 5

114

xxx e)

222 3

5

3

11

k

k

kk

f)

xxx

x 6152

g) kkkk

1

6

162

h) 6

1

65

1

1

42

nnnn

i) xxxxx 5

4

5

1

5

122

j) ppppp 6

2

6

1

6

522

12.6.5 Resol les següents equacions:

a) vv

v

vv

v

v 122

6

6

155

2

122

b)

3

1

32

6

1

52

xxxx

c) 22

2

6

1

6

167

nn

nn

d)

k

kk

k

k

4

431

1 2

e) yy

121

2 f)

n

n

n

nn

5

61

5

1082 2

g) 2223

2 2143

x

x

xxx

xx

h)

43

3

1

21

2

nnn

n

i) 2

1

22422

62

v

v

vv

v j)

102

183122

102

3 2

x

xxx

x

x

Page 70: TEORÍA DE NÚMEROS1 Principios. Problemas sin teoría. La "Teoría de números" o "aritmética" estudia las propiedades de los números enteros. Los conceptos teóricos de esta rama

13 Expressions i equacions amb radicals.

13.1 El domini de definició de les expressions amb radicals.

Una expressió )(xp no està definida si 0)( xp

Una expressió 3 )(xp sempre està definida.

La presencia d'una arrel quadrada porta implícita la condició de què el seu interior no és

negatiu. Per exemple, l'expressió 3x només té sentit si 3x .

13.1.1 Indica el domini de definició de les següents expressions:

a) 5x b) 8y c) 33 a d) 65 m e) 82 x

Page 71: TEORÍA DE NÚMEROS1 Principios. Problemas sin teoría. La "Teoría de números" o "aritmética" estudia las propiedades de los números enteros. Los conceptos teóricos de esta rama

13.2 Equacions amb radicals senzilles.

Les equacions BA i 22 BA no són equivalents: La segona té dues solucions: BA i

BA , una més que la primera. Per tant, quan elevem al quadrat una equació poden aparèixer

solucions no vàlides.

No és cert que AA 2 . Estrictament parlant, la igualtat és la següent: AA 2.

Per exemple: 3)3( 2 , en realitat: 3)3( 2 .

És cert que AA 2

, i aquesta expressió només té sentit si 0A , perquè si 0A no té

sentit A , i per tant tampoc té sentit 2A .

Les equacions BA i 33 BA sí son equivalents.

En general, les equacions amb arrels d'índex parell poden generar solucions no vàlides, i les

equacions amb arrels d'índex senar, no.

Tota aquesta problemàtica es redueix a seguir una única norma:

És obligatori comprovar totes les solucions quan resolem equacions amb

radicals, perquè no totes les solucions que obtinguem seran sempre vàlides.

Exercici resolt.

Resol l'equació 75 x

Solució:

44549497575 2 xxx

Comprovació: 749544

13.2.3

Resol les següents equacions:

a) 10x b) 2

1x c) 4x d) 0x e) 53 x

f) 12 x g) 3 x h) 3 x

13.2.4

Resol les següents equacions:

a) 0532 x b) 0532 x c) 7253 x

d) 2562

x

e) 3132 x f) 0532 x

g) 15543 x h) 53

24 x

Page 72: TEORÍA DE NÚMEROS1 Principios. Problemas sin teoría. La "Teoría de números" o "aritmética" estudia las propiedades de los números enteros. Los conceptos teóricos de esta rama

Exercici resolt.

Resoldre l'equació: xx 313191

Solució:

Aïllem l'arrel quadrada:

131319 xx

Elevem al quadrat i resolem l'equació resultant:

9/7

20142591691319131319 2222

x

xxxxxxxx

Comprovem les solucions:

651625161338123132191

3

7

3

7

3

7

3

41

3

79/161

9

7313

9

7191

Les dues solucions són acceptables. Les solucions són 2x i 9

7x

13.2.1

Resol les següents equacions:

a) 62632 xxx b) 510 x c) 67 a

d) 284 xx e) 11832 x f) aa 552

g) 2545 b

Exercici resolt.

Resol l'equació 05283 aa

Solució:

385235283

528352830528322

aaaaa

aaaaaa

Comprovació:

1156895)3(28)3(3

No té sentit, perquè no existeixen arrels de nombres negatius. Per tant, aquesta equació

no té solució.

Page 73: TEORÍA DE NÚMEROS1 Principios. Problemas sin teoría. La "Teoría de números" o "aritmética" estudia las propiedades de los números enteros. Los conceptos teóricos de esta rama

13.2.2

Resol:

a) 1546 mm b) 7365 xx

c) 18367 aa d) 092710 aa

e) 0109512 kk f) 0836 xx

g) 020754 aa h) 0492 aa

Exercici resolt.

Resol l’equació: )1(352 xx

Solució:

Elevem al quadrat tots dos costats:

222

)1(952)1(352)1(352 xxxxxx

Resolem l'equació que en resulta, que ja no té arrels quadrades:

9/2

2042090918952

91895212952)1(952

22

222

x

xxxxxx

xxxxxxxx

Comprovem aquests dos possibles resultats:

Per a 2x 39)12(3522 2 x és vàlida.

Per a 9/2x3

7

3

71

9

235

9

22

9

2 x no és vàlida.

L'única solució és 2x .

Exercici resolt.

Resol l’equació: 2315 xx

Solució:

224

484824252325

232525231252315

xxxx

xxxxxx

Tanmateix, 2x no és una solució vàlida, perquè si substituïm a l'equació original

apareixen arrels de valors negatius, que no existeixen:

2515232125

Per tant aquesta equació no té solució.

13.2.5 Resol les següents equacions:

a) xx 12 b) xx 43 c) xx 45

d) xx 225

Page 74: TEORÍA DE NÚMEROS1 Principios. Problemas sin teoría. La "Teoría de números" o "aritmética" estudia las propiedades de los números enteros. Los conceptos teóricos de esta rama

13.2.6

Resol les següents equacions:

a) 2162 xx b) 142 xx c) 112 xx

d) 1254 2 xx e) 0125.02 2 xxx

13.2.7 Resol les següents equacions:

a) 21 x b) 21 x c) xx 21341

d) xx 1352 e) xx 192 f) xx 31152

Page 75: TEORÍA DE NÚMEROS1 Principios. Problemas sin teoría. La "Teoría de números" o "aritmética" estudia las propiedades de los números enteros. Los conceptos teóricos de esta rama

13.3 Equacions amb sumes de radicals.

Exercici resolt.

Resoldre l'equació 32716457128 xx

Solució:

Aïllem la primera arrel:

7164325712832716457128 xxxx

Elevem al quadrat:

222

7164325712871643257128 xxxx

Desenvolupem el quadrat de la dreta:

xxxx

xxx

6471646495371647164641024

71647164322327164322

22

L'equació queda: xxx 6471646495357128

I tornem a començar! Aïllem l'arrel quadrada:

xx

xxx

64896716464

5712864953716464

La simplifiquem dividint entre 64:

xxxx 14716464896716464

Elevem al quadrat:

222

147164147164147164 xxxxxx

Desenvolupem el quadrat de la dreta:

2222281961421414 xxxxx

L'equació queda:

89

3

)1(2

8692

)1(2

)267()1(49292026792

0281967164

281967164

22

2

2

xxx

xxx

xxx

Comprovem les solucions obtingudes:

321121121441713645731283 xSi

7510756251144971896457891283xSi

32187

Per tant, l'única solució d'aquesta equació és 3x .

Page 76: TEORÍA DE NÚMEROS1 Principios. Problemas sin teoría. La "Teoría de números" o "aritmética" estudia las propiedades de los números enteros. Los conceptos teóricos de esta rama

Exercici resolt.

Resoldre l'equació 4117 xx

Solució:

)1(181617

11424171417

14174117

22

22

xxx

xxxxx

xxxx

Ara ja només hi queda una arrel quadrada. Tornem a començar! L'aïllem i elevem al

quadrat:

9/13

5065589

16164942911649429

116731473

181461811617

2

22

22

x

xxx

xxxxxx

xxxx

xxxxx

El primer resultat és vàlid: 42643615157

Però el segon no ho és: 43

8

3

2

3

10

9

4

9

1001

9

131

9

137

Exercici resolt.

Resol l'equació 43132 xxx

Solució:

Elevem al quadrat:

4313243132222

xxxxxx

Desenvolupem el quadrat de l'esquerra:

)1)(32(2431)1)(32(232

1132232132222

xxxxxxx

xxxxxx

Recuperem l'equació i simplifiquem:

02

0)1)(32(0)1)(32(243)1)(32(243 xxxxxxxx

Una arrel quadrada només pot ser zero si el seu interior és zero:

0)1)(32(0)1)(32( xxxx

Un producte només pot ser zero si algun dels seus factors és zero:

Page 77: TEORÍA DE NÚMEROS1 Principios. Problemas sin teoría. La "Teoría de números" o "aritmética" estudia las propiedades de los números enteros. Los conceptos teóricos de esta rama

101

2

3032

0)1)(32(

xx

xxxx

Verifiquem aquests resultats amb l'equació original:

2

1

2

1

2

1

2

104

2

331

2

33

2

32

2

3xSi

101413113121 xSi

Aquest resultat queda fora del domini de definició de l'equació, per tant no és vàlid.

L'única solució de l'equació és 2

3x .

13.3.1 Resol les següents equacions:

a) 553)1(323)1(3 xxxx b) 33234 xx

c) xxx 35656 d) 48283 22 xx

e) 252512 22 xxx f) 382238 xx

g) xxxx 3316)13( 2

h) xxxxx 5122)22)(22( 2

13.3.2 Resol les següents equacions:

a) xx 33 b) xx 2554 c) 12 xx

d) 1125 xx e) 2265 xx

13.3.3

Resol les següents equacions:

a) xxx 547 b) 3322 xx

c) 141 xxx d) 12352 xx

e) 1924125 xxx f) 49145 xxx

13.3.4

Resol les següents equacions:

a) 453 22 xx b) xxx 253 22

c) 7352 xxx

Page 78: TEORÍA DE NÚMEROS1 Principios. Problemas sin teoría. La "Teoría de números" o "aritmética" estudia las propiedades de los números enteros. Los conceptos teóricos de esta rama

13.3.5 Resol les següents equacions:

a) 62632 xxx b) xxxx 2

1123

c) 332262 2 xxx d) xxxx 23255252

e) xxx 317

213

5

1 f) 4

2

162214

2

1 xxx

g) 5

3

5

11

5

316

4

5

2

15 2

xxxx

h) 273654322 2 xxxxx

13.3.6

Resol aquesta equació racional-irracional: 33

52

x

x

xx

13.6.7 Resol les següents equacions:

a) 192534 xxx

b) 4213267 xxxx

c) xxxx2

9

2

34

2

1

2

128

d) 4

3

4

32122 xxx

e) 3

1032

3

1036

3

7

3

1 xxxx

f)

12

151

3

54

4

1xxx

g) )1(423178 xxxx

Page 79: TEORÍA DE NÚMEROS1 Principios. Problemas sin teoría. La "Teoría de números" o "aritmética" estudia las propiedades de los números enteros. Los conceptos teóricos de esta rama

14 Equacions exponencials i logarítmiques.

14.1 El domini de definició d'expressions exponencials i logarítmiques.

Una expressió exponencial )(xpa sempre està definida.

Una expressió logarítmica )(log xpn no està definida si 0)( xp

Els logaritmes es comporten com les arrels quadrades: No estan definits per a

valors negatius, però amb una diferència: Existeix 0 però no existeix )0(logn .

14.2 Equacions exponencials igualant bases.

Si les bases són iguals sempre les podem "tatxar":

cbaa cb

Exercici resolt.

Resol l'equació 644 2 x

Solució:

13244644 322 xxxx

Exercici resolt.

Resol l'equació 13 84 xx

Solució:

1)3(36

22

22

84

)3(36

1332

13

xxx

xx

xx

xx

14.2.1 Resol les següents equacions:

a) 14 32 x b) xx 55 23 c) 2433 21 x d) aa a23

e) 14 23 x f) 122 44 pp g) aa 322 66 h) xx 322 22

14.2.2

Page 80: TEORÍA DE NÚMEROS1 Principios. Problemas sin teoría. La "Teoría de números" o "aritmética" estudia las propiedades de los números enteros. Los conceptos teóricos de esta rama

Resol les següents equacions igualant les bases i aplicant les propietats dels exponents:

a) mmm 23 666 b) x

x

x22

2

2 c) 10

11010 3 xx

d) xxx 333 3212 e) 6444 2 xx f) 216

166 2 xx

g) 3232

12 x h) ppp 223 222 i) 233 161664 xx

j) 423

3243

81

n

n

k) 27981 22 b l) 2799 3 xx

m) 216

1216

6

1 3

23

x

x

n) 99243 122 kk o) 646416 33 xx

p) 4232 2416 pp

Page 81: TEORÍA DE NÚMEROS1 Principios. Problemas sin teoría. La "Teoría de números" o "aritmética" estudia las propiedades de los números enteros. Los conceptos teóricos de esta rama

14.3 Equacions exponencials resoltes amb logaritmes.

Si les bases són diferents, aïllem l'exponent amb un logaritme:

cbca a

b log

Exercici resolt.

Resol l'equació 72 x

Solució:

807.27log72 2 xx

Exercici resolt.

Resol l'equació 205 1 xe

Solució:

465.1115log15log115352032035 33

111 xxxxx

Exercici resolt.

Resol l'equació 21410 32 x

Solució:

115.23

3)17(log

3)17(log2

)17(log32

1742110

21410

10

10

10

32

32

x

x

x

x

x

Page 82: TEORÍA DE NÚMEROS1 Principios. Problemas sin teoría. La "Teoría de números" o "aritmética" estudia las propiedades de los números enteros. Los conceptos teóricos de esta rama

14.4 Equacions exponencials mitjançant canvi de variable.

Exercici resolt.

Resol l'equació 115113532 xx

Solució:

Fem el canvi de variable xz 3 , i resolem l'equació de segon grau que ens queda:

9

141151152

z

zzz

Desfem el canvi de variable: xz 314 no té solució perquè una exponencial mai és negativa.

239 xz x

Comprovem que aquesta possible solució sigui vàlida:

1151151151145811151145311511353 4222

Per tant, la única solució és 2x

14.4.1 Resol:

a) 01222 24 xx b) 081369 1 xx c) 3033 2 xx

d) 5

31555 11 xxx

14.4.2

Resol:

a) 03252 1 xx b) 0232 xx ee

c) 07293909 xx d) 0542 xx ee

e) 021664236 xx f) 4822 2 qq

g) 7222 21 xxx h) 9602222 321 xxxx

i) 032042 2 xx j) 093183 )1(2 xx

k) 081329 31 xx l) 081329 31 xx

m)

x

x

2

14 3 n) 21633 11 22

xx

o) 2

384

x

x p) 124

1

525

2

xx

Page 83: TEORÍA DE NÚMEROS1 Principios. Problemas sin teoría. La "Teoría de números" o "aritmética" estudia las propiedades de los números enteros. Los conceptos teóricos de esta rama

14.5 Equacions logarítmiques senzilles.

"Passem a l'altra banda" el logaritme amb una exponencial

c

a abcb log

Atenció! Tenim molt en compte que ara hi ha domini de definició: 0b i poden

aparèixer solucions no vàlides. Les solucions s'han de comprovar!

Exercici resolt.

Resol la següent equació: 5)75(log2 x

Solució:

532752755)75(log 5

2 xxxx

Comprovem el resultat: 555)32(log5)755(log 22

14.5.1 Resol les següents equacions:

a) 5log3 x b) 3log4 x c) 100log2log 22 x

d) 7ln)4ln( x e) 212log3 x f) 210log5 x

g) 2115log x

14.5.2

Resol les següents equacions:

a) 95log37log 33 xx b) 14log)3)(2(log 77 xx

Page 84: TEORÍA DE NÚMEROS1 Principios. Problemas sin teoría. La "Teoría de números" o "aritmética" estudia las propiedades de los números enteros. Los conceptos teóricos de esta rama

14.6 Equacions amb sumes de logaritmes.

Hem d'aplicar la propietat:

)(log)(log)(log baba nnn

Exercici resolt.

Resol l'equació: 3)2(log)(log 22 xx

Solució:

4

208282)2(

3)2(log3)2(log)(log

23

222

x

xxxxx

xxxx

Comprovem les possibles solucions:

3)22(log)2(log 22 no té sentit perquè no existeixen logaritmes de nombres

negatius.

3123)24(log)4(log 22 aquesta solució sí és vàlida.

Per tant, l'única solució és 4x .

14.6.1

Resol les següents equacions:

a) 1)1(loglog 66 xx b) 1)2(log)3(log 22 xx

c) 1)3(log)2(log 66 xx d) 3)12(loglog 44 xx

e) )4(log)2(log)4(log 666 xxx

Page 85: TEORÍA DE NÚMEROS1 Principios. Problemas sin teoría. La "Teoría de números" o "aritmética" estudia las propiedades de los números enteros. Los conceptos teóricos de esta rama

14.7 Equacions amb restes de logaritmes.

Hem d'aplicar la propietat:

b

aba nnn log)(log)(log

Exercici resolt.

Resol l'equació: )3ln()3ln()45ln( xx

Solució:

439)3(345

33

45)3ln(

3

45ln)3ln()3ln()45ln(

xxxx

x

x

x

xxx

Comprovem la solució: )3ln()7ln()21ln()3ln()43ln()445ln(

)3ln(3ln)3ln(7

21ln

14.7.1 Resol les següents equacions:

a) 4log)5(log)8(log 777 xx b) 11log)9(log)7(log 222 xx

c) 0)11(log)53(log 55 xx d) 0)5(log)12(log 66 xx

e) )10(log3)3(log 22 xx f) )1(log2)815(log 33 xx

g) 3log)2(log)12(log 444 xx h) 3)4(log)1(log 22 xx

14.7.2 Resol les següents equacions:

a) )3log(5log)4log(3log xx b) )3ln(9ln)8ln(6ln xx

14.7.3 Resol les següents equacions:

a) )12log(2loglog2 xx b) )8log(4loglog2 xx

14.7.4 Repàs d’equacions logarítmiques:

a) 2)35log( x b) 3)2(loglog 22 xx

c) 2)2(log)(log2 44 xx d) 0)9ln()2ln(4 2 xx

e) 1)6log()3log( xx f) 1log)53log( xx

Page 86: TEORÍA DE NÚMEROS1 Principios. Problemas sin teoría. La "Teoría de números" o "aritmética" estudia las propiedades de los números enteros. Los conceptos teóricos de esta rama

Exercici resolt.

Resol l'equació: )log()log( xx

Solució:

10101)log(01)log(

1100)log(01)log()log(

0)log()(log)log()(log)log()log(

1

0

22

xxx

xxxx

xxxxxx

Les dues solucions són vàlides:

00)1log()1log( 11)10log()10log(

Page 87: TEORÍA DE NÚMEROS1 Principios. Problemas sin teoría. La "Teoría de números" o "aritmética" estudia las propiedades de los números enteros. Los conceptos teóricos de esta rama

14.8 Equacions logarítmiques amb productes.

Hem d'aplicar la propietat:

n

aa bbn loglog

En particular: n

a

n

aaa bbb

nn

blogloglog

1log /1

14.8.1 Resol les següents equacions:

a) 1)6log(log2 2 xx b) )4log(2

12log)45log( xx

c) xx

log24

625log

5log4

d) 0)4log(325log 3 xx

e)

35log

35log 3

x

x f) xx 5log211log2log 2

g) 9.0log1log2 xx h)

2log32loglog3

xx

i) )12log()2log()log(2 xx j) )8log()4log()log(2 xx

k) 2)43log(

)16log( 2

x

x

14.8.2 Repàs d’equacions exponencials.

a) 3 93 x b) 24339 1 x c) 822 1 xx

d) 012552 xx e) 322 1 xx f) 16

1728 131 xx

g) 042522 xx h) 0639 xx i) 077507 21 xx

14.8.3 Repàs d’equacions logarítmiques.

a) 5

4)35log( x b) 3)2(loglog 22 xx

c) 2)2(log)(log2 44 xx d) 0)9ln()2ln(4 2 xx

e) 1)6log()3log( xx f) 1log)53log( xx

14.8.4 Repàs d'equacions exponencials i logarítmiques.

a) 34 x b) 2)(log3 x

c) 4254 x d) 4324 53 x

e) 3)14(log)2(log 55 xx f) 2)34(log53 2 x

g) )5log()42log()23log( xx h) )4log()32log()24log( xx

i) )45log()14log()34log(2 xxx j) 09437 2 xx

Page 88: TEORÍA DE NÚMEROS1 Principios. Problemas sin teoría. La "Teoría de números" o "aritmética" estudia las propiedades de los números enteros. Los conceptos teóricos de esta rama

14.9 Problemes amb equacions exponencials i logarítmiques.

14.9.1

Els valors de a en l'equació 215log 2

10 aa són:

(A) 2

23315 (B) 5,10 (C)

2

30515 (D) 20

(E) Cap dels anteriors.

AHSME 1951, #22

Solució: PA/#1.7

Page 89: TEORÍA DE NÚMEROS1 Principios. Problemas sin teoría. La "Teoría de números" o "aritmética" estudia las propiedades de los números enteros. Los conceptos teóricos de esta rama

15 Inequacions amb una incògnita.

Una inequació és una desigualtat (el signe de la qual pot ser <, >, ≤ i ≥) entre

expressions algebraiques. Per exemple 1273 xx és una inequació.

Com en el cas de les equacions, las incògnites de cada membre d’una inequació es

poden substituir, també, per valors numèrics. Per exemple, en la inequació anterior es

poden substituir la x per 1:

1273

112713

34

D’aquesta manera, la inequació es transforma en una desigualtat entre expressions

numèriques. En cas que sigui certa, es diu que s’ha trobat una solució de la inequació.

Així, una solució de la inequació 1273 xx és 1x perquè acabem de veure que

satisfà la inequació.

Una inequació pot tenir més d’una solució. Per exemple, en el cas de la inequació

anterior, altra solució podria ser x = 2:

1476

122723

51

En general, les inequacions donen com a solució tot un interval, que pot ser obert o

tancat, finit o infinit. Per exemple, la solució de la inequació anterior és l'interval 8x .

15.1 Inequacions de primer grau amb una incògnita.

Resolució d'inequacions de primer grau per transformacions equivalents.

Dues inequacions que tenen les mateixes solucions es denominen equivalents. Es pot

trobar una inequació equivalent a una altra utilitzant procediments similars als coneguts

per a les equacions:

- Sumant o restant a ambdós membres el mateix nombre.

- Multiplicant o dividint ambdós membres pel mateix nombre (diferent de 0). En aquest

cas, cal destacar que si el factor pel qual es multipliquen (o es divideixen) ambdós

membres és negatiu, llavors el signe de la desigualtat canvia d’orientació (és a dir, <

es transforma en >, i > es transforma en <).

Per exemple, una inequació equivalent a l’equació xx 243 es pot obtenir

multiplicant ambdós membres per –2, i és:

xxxx 2)2(43)2(243

és a dir:

xx 2486

Això és així perquè és sabut que en multiplicar (o dividir) ambdós membres d’una

desigualtat per un nombre negatiu, la desigualtat ha de canviar la seva orientació.

Page 90: TEORÍA DE NÚMEROS1 Principios. Problemas sin teoría. La "Teoría de números" o "aritmética" estudia las propiedades de los números enteros. Los conceptos teóricos de esta rama

Resolució d'inequacions de primer grau representant la gràfica de la recta.

Exercici resolt.

Resol la inequació: 053 x

Solució:

Trobem els punts de tall de la funció 53)( xxf amb l'eix X:

66.13

5053)( xxxf

La funció és creixent perquè el seu coeficient principal és positiu. Dibuixem la seva

gràfica:

Està clar que la funció serà negativa, és a dir, 053 x a l'esquerra del punt de tall:

3

5053 xx

I per tant l'interval solució de la inequació serà

3

5,

Page 91: TEORÍA DE NÚMEROS1 Principios. Problemas sin teoría. La "Teoría de números" o "aritmética" estudia las propiedades de los números enteros. Los conceptos teóricos de esta rama

Resolució d'inequacions de primer grau amb punts frontera i punts de prova.

Per a resoldre una inequació de primer grau s’han de seguir aquests passos:

1) Es resol l’equació associada a la inequació lineal. L’equació associada a una

inequació és aquella que s’obté canviant el signe de desigualtat pel signe igual. Aquesta

solució, diguem a, s'anomena punt frontera, i divideix la recta real en dos intervals:

a, i ,a

2) Es tria un nombre, qualsevol, que no sigui el punt frontera a, al que direm punt de

prova.

3) Se substitueix aquest nombre a la inequació, i es comprova si és solució.

4) Si és solució, la solució de la inequació serà l'interval al què pertany el punt de prova.

Si no, serà l'altre. Llegeix detingudament l'exemple següent:

Exercici resolt.

Resol la inequació: 135 x

Solució:

Resolem l'equació associada: 33.13

4315135 xxx

Aquest valor s'anomena punt frontera perquè separa la recta en dos intervals:

3

4, i

,

3

4

Un d'ells, i només un d'ells, és el bo. Per saber quin dels dos és el correcte prenem un

punt de prova, qualsevol valor que no sigui el punt frontera, per exemple 0x , i

comprovem si la inequació original es verifica amb aquest valor:

1510350 x

Com que la inequació es verifica amb el punt de prova 0x , també es verificarà amb

tots els valors de l'interval on està el punt frontera:

0x pertany a l’interval 3/4,

Que és el conjunt solució de la inequació: 3/4,

Exercici resolt.

Resol la inequació xx 3732

Solució:

Trobem el valor frontera resolent l'equació corresponent:

25

1010537323732 xxxxxx

Agafem qualsevol valor que no sigui 2, per exemple el 0, i mirem si satisfà la inequació

inicial:

73037302

Per tant, l'interval solució serà aquell que agafa el 0, és a dir, 2,

15.1.1 Resol les següents inequacions:

Page 92: TEORÍA DE NÚMEROS1 Principios. Problemas sin teoría. La "Teoría de números" o "aritmética" estudia las propiedades de los números enteros. Los conceptos teóricos de esta rama

a) 712 x b) xx 5863 c) xx 231

d) xx 374 e) 135 xx

15.1.2

Resol les següents inequacions.

a) 713 x b) 4010 x c) 156 x

d) 32)1(4 xx e) )4(372 xx f) 764

3 xx

g) 27

23 xx h) 13)2(31 x i) 5

4

32

x

j) 3

1

2

3

xx

15.1.3 Resol les següents inequacions:

a) 3417 xx b) 1253 x

c) 735 yy d) )35(2)13(2 xxx

e) xx 13

2 f) 2)23(5 a

g) 14)1(325 xxx h) 52)2(3 xx

i) 12)1(23 xxx j) 1213 ww

k) 3

12

2

1352

xxx l) 83)1(3)1(5 xxx

m) )1(2)8(5)4(3 xxx n) 3

7

5

1362

xxx

o) xxx10

3)4(

2

1)1(

5

2 p) x

xx

3

3

48

4

13

q) 1625

2

x

xx r)

2

32

3

12 xx

x

15.1.4 Resol les següents inequacions (que semblen de segon grau però són de primer):

a) )3(2)32)(32()2(3)12( 2 xxxxx

b) 32)56(3)13(2)15(4 2 xxxxx

c) )43(2)2(4 xxxxx

d) )1(12)2(4)12()12(4 22 xxxxx

e) )21)(12()21(5)2()2( 32 xxxxxx

f) 2233 )2(2)1)(1(2622)1()1( xxxxxxx

Page 93: TEORÍA DE NÚMEROS1 Principios. Problemas sin teoría. La "Teoría de números" o "aritmética" estudia las propiedades de los números enteros. Los conceptos teóricos de esta rama

15.1.5

Resol les següents inequacions:

a) 23)1(3 x b) xxx 37)2(35

c) )32(3)5(24 xx d) 1372 22 xxx

15.1.6 Resol les següents inequacions:

a) 8)2(23 xx b) )63(3)2(5 xx

c) )3(25)23(6 xx d) 4)35(367)53(2 xxx

Page 94: TEORÍA DE NÚMEROS1 Principios. Problemas sin teoría. La "Teoría de números" o "aritmética" estudia las propiedades de los números enteros. Los conceptos teóricos de esta rama

15.2 Inequacions de segon grau amb una incògnita.

Resolució d'inequacions de segon grau representant la gràfica de la paràbola.

Exercici resolt.

Resol la inequació: 0432 xx

Solució:

Trobem els punts de tall de la funció 43)( 2 xxxf amb l'eix X:

1

4

12

253

12

)4(14)3(3043

22

x

xxxx

La paràbola té les branques cap a dalt perquè el coeficient principal és positiu. Per tant,

la seva gràfica és:

Veient la gràfica deduïm clarament que 410432 xoxxx

És a dir, el conjunt solució és ,41,

Page 95: TEORÍA DE NÚMEROS1 Principios. Problemas sin teoría. La "Teoría de números" o "aritmética" estudia las propiedades de los números enteros. Los conceptos teóricos de esta rama

Exercici resolt.

Resol la inequació: 0442 xx

Solució:

Trobem els punts de tall de la funció 44)( 2 xxxf amb l'eix X:

2

2

12

04

12

414)4(4044

22

x

xxxx

Aquesta paràbola només té un punt de tall amb l'eix X, i té les branques cap a dalt

perquè el coeficient principal és positiu. Per tant, la seva gràfica és:

Observem que tota la funció està per sobre de l'eix X, per tant 0442 xx sempre

que 2x

El conjunt solució és ,22,2IR

15.2.1 Resol les següents inequacions:

a) 062 xx b) 0322 xx

Page 96: TEORÍA DE NÚMEROS1 Principios. Problemas sin teoría. La "Teoría de números" o "aritmética" estudia las propiedades de los números enteros. Los conceptos teóricos de esta rama

15.2.2

Només observant la gràfica de la funció 34)( 2 xxxf , sense fer cap càlcul, resol

a) 0342 xx b) 0342 xx c) 0342 xx

d) 0342 xx

15.2.3

Només observant la gràfica de la funció 44)( 2 xxxf , sense fer cap càlcul, resol

a) 0442 xx b) 0442 xx c) 0442 xx

d) 0442 xx

Page 97: TEORÍA DE NÚMEROS1 Principios. Problemas sin teoría. La "Teoría de números" o "aritmética" estudia las propiedades de los números enteros. Los conceptos teóricos de esta rama

Resolució d'inequacions de segon grau amb punts frontera i punts de prova.

Seguirem el mateix procediment que amb les inequacions de primer grau. Ara treballem

amb paràboles, i per tant, en principi, tindrem dos punts frontera: a i b , que

obtindrem resolent l'equació corresponent.

Exercici resolt.

Resol la inequació 01072 xx

Solució:

Resolem l'equació associada: 01072 xx :

5

2

2

97

)1(2

)10()1(4770107

22

x

xxxx

Dibuixem una recta i marquem els punts obtinguts, anomenats "punts frontera".

Prenem punts de prova entre els punts frontera. Per exemple: 0x , 3x i 7x

Provem la inequació inicial amb els punts de prova:

0100100700 2 x

020102190103733 2 x

01001049490107777 2 x

Els intervals solució seran aquells on els punts de prova han donat cert:

,52,

Tot plegat és més fàcil si t'imagines la gràfica de la funció:

Page 98: TEORÍA DE NÚMEROS1 Principios. Problemas sin teoría. La "Teoría de números" o "aritmética" estudia las propiedades de los números enteros. Los conceptos teóricos de esta rama

15.2.4 Resol les següents inequacions:

a) 016102 xx b) 0122 xx c) 01032 xx

15.2.5 Resol les següents inequacions:

a) 972 2 xx b) 1042 xx c) xxx 101232

d) 13211122 22 xxxx e) 64535 22 xxxx

15.2.6 Resol les següents inequacions:

a) 0232 xx b) 062 xx c) 016 2 xx

d) 0822 xx e) 0232 xx f) 0342 xx

g) 0862 xx h) 08

15

4

72 xx i) 016

132 xx

15.2.7

Resol les següents inequacions:

a) )1(2

12 xx b) 06)1(25 xx c) )1(94 xx

d)

2

3

2

1

2

12

xx e) 2)2(8

1352

xx

f) )74(2)1(2 xxx

Page 99: TEORÍA DE NÚMEROS1 Principios. Problemas sin teoría. La "Teoría de números" o "aritmética" estudia las propiedades de los números enteros. Los conceptos teóricos de esta rama

Inequacions de segon grau amb discriminat nul.

Si l'equació de segon grau associada a la inequació té discriminant nul, el conjunt

solució pot ser el conjunt buit, tota la recta real o un únic punt. És molt important

dibuixar la gràfica de la paràbola corresponent i tenir molt en compte si és una

desigualtat estricta (amb o ) o no estricta (amb o ).

Exercici resolt.

Resol la inequació 025102 xx

Solució:

L'equació associada té discriminant nul: 01001002514)10( 2 .

La seva única solució és 52

10

2

010

x

Es tracta d'una paràbola amb les branques cap a dalt que passa pel punt )0,5(

Mirant la gràfica veiem que només per a 5x podem satisfer 025102 xx , per

tant, la solució és 5x .

15.2.8 Resol les següents inequacions:

a) 01022 xx b) 03

443 2 xx c) 011881 2 xx

d) 0145484 2 xx e) 092416 2 xx f) 0542 xx

g) 08

8192 2 xx h) 034102 xx i) 02082 xx

j) 025309 2 xx

Page 100: TEORÍA DE NÚMEROS1 Principios. Problemas sin teoría. La "Teoría de números" o "aritmética" estudia las propiedades de los números enteros. Los conceptos teóricos de esta rama

15.3 Inequacions de tercer grau i superior amb una incògnita.

El mètode de "Punts frontera+Punts de prova" que hem estudiat per resoldre les

inequacions de primer i segon grau es pot aplicar a la resolució de inequacions de graus

superiors a dos.

Exercici resolt.

Resol la inequació 483232 23 xxx

Solució:

Determinem els punts frontera resolent l'equació polinòmica associada:

0483232483232 2323 xxxxxx

La resolem amb el mètode de Ruffini:

2 -3 -32 48

-4 -8 44 -48

2 -11 12

-12

0

4 8

2 -3 0

3/2 3

2 0

Els punts frontera són 4x , 5.12

3x , 4x .

Dividim la recta en tres intervals amb aquests punts frontera i prenem valors de prova

dintre de cada interval:

A l'interval 4, prenem, per exemple, 5x :

4816548160253)125(248)5(32)5(3)5(2 23

A l'interval

2

3,4 prenem, per exemple, 0x :

480480320302 23

A l'interval

4,

2

3 prenem, per exemple, 2x :

4860482324382482322322 23

A l'interval ,4 prenem, per exemple, 5x :

4815485322531252485325352 23

El conjunt solució de la inequació serà la unió dels intervals on s'ha verificat el punt de

prova:

,4

2

3,4

Page 101: TEORÍA DE NÚMEROS1 Principios. Problemas sin teoría. La "Teoría de números" o "aritmética" estudia las propiedades de los números enteros. Los conceptos teóricos de esta rama

15.3.1 Resol les següents inequacions de tercer grau:

a) 03272 23 xxx b) 02323 23 xxx

c) 04488 23 xxx d) 0122 23 xxx

e) 015133 23 xxx f) 0573 23 xxx

15.3.2 Resol les següents inequacions:

a) 0)1)(1()1(2 2 xxxx b) 0)15(6)111(3 3 xxx

15.3.3

Inequacions biquadrades. Resol les següents inequacions:

a) 0161459 24 xx b) 025376 42 xx

15.3.4

Resol les següents inequacions.

a) 03612112 234 xxxx b) 012 234 xxxx

c) 0184116 24 xx

Page 102: TEORÍA DE NÚMEROS1 Principios. Problemas sin teoría. La "Teoría de números" o "aritmética" estudia las propiedades de los números enteros. Los conceptos teóricos de esta rama

16 Sistemes d'equacions lineals amb dues incògnites.

16.1 Els tres mètodes: Substitució, igualació i reducció.

Mètode de substitució.

El mètode de substitució consisteix a aïllar una de les incògnites d’una de les dues

equacions i substituir el seu valor en l’altra equació. Una vegada resolta aquesta última

(que només tindrà una única solució), es resol l’altra equació introduint aquest valor.

Exemple resolt.

Resol el sistema

42

824

yx

yx

Solució:

1. Es tria una de les dues equacions.

42 yx

2. S’aïlla una de les incògnites de l’equació.

xy 24

3. Se substitueix el valor de la incògnita en l’altra equació.

8)24(24 xx

4. Es resol l’equació resultant.

2

8/16

168

888

8484

x

x

x

x

xx

5. Se substitueix aquesta incògnita en l’equació del pas 2, pel valor trobat.

0224 y

La solució és: x = 2 i y = 0

S’ha de comprovar la solució:

4022

80224

Mètode d'igualació.

Page 103: TEORÍA DE NÚMEROS1 Principios. Problemas sin teoría. La "Teoría de números" o "aritmética" estudia las propiedades de los números enteros. Los conceptos teóricos de esta rama

El mètode d’igualació consisteix a aïllar la mateixa incògnita d’ambdues equacions i igualar

els resultats obtinguts. Una vegada resolta aquesta equació, se’n pot substituir el valor en una

de les equacions inicials i resoldre-la per a trobar l’altre valor.

Exercici resolt.

Resol el sistema

42

824

yx

yx

Solució:

1. S’aïlla la mateixa incògnita en totes dues equacions.

xy

xy

24

2

84

2. S’igualen les dues expressions resultants.

xx

242

84

3. Es resol l’equació resultant.

28

161688844

4884)24(284242

84

xxxx

xxxxxx

4. Es substitueix la incògnita de qualsevol de les equacions del sistema del pas 1 pel

valor trobat.

0224 y

La solució és: x = 2 i y = 0

5. S’ha de comprovar la solució.

4022

80224

Page 104: TEORÍA DE NÚMEROS1 Principios. Problemas sin teoría. La "Teoría de números" o "aritmética" estudia las propiedades de los números enteros. Los conceptos teóricos de esta rama

Exercici resolt.

Resol el següent sistema

245

523

yx

yx amb el mètode d'igualació.

Solució:

Aïllem la variable x a les dues equacions:

5

42425245

3

25253523

yxyxyx

yxyxyx

Igualem les expressions obtingues:

5

42

3

25

5

42

3

25

yy

yx

yx

Resolem l’equació obtinguda :

2

19

2

1919225612101261025

)42(3)25(55

42

3

25

yyyyyy

yyyy

Substituïm el valor de la incògnita obtinguda:

83

24

3

195

3

2/1925

3

25

yx

Per tant, la solució al sistema és 2

19,8 yx .

Efectivament, comprovem que les solucions proposades satisfan les equacions

519242

19283 23840

2

19485

Page 105: TEORÍA DE NÚMEROS1 Principios. Problemas sin teoría. La "Teoría de números" o "aritmética" estudia las propiedades de los números enteros. Los conceptos teóricos de esta rama

Mètode de reducció.

El mètode de reducció consisteix a multiplicar convenientment ambdues equacions de manera

que una vegada restades, desaparegui una de les incògnites. Una vegada resolta l’equació

resultant, es pot substituir aquest valor en una de les equacions inicials i resoldre-la per a

obtenir la solució general.

Exercici resolt.

Resol el sistema

42

824

yx

yx

Solució:

1. Es tria una de les dues incògnites.

es tria la y

2. Es multipliquen els dos membres de la primera equació pel coeficient de la incògnita

escollida en la segona equació, i els dos membres de la segona equació pel coeficient de

la incògnita escollida en la primera equació.

422

8241

yx

yx

824

824

yx

yx

3. Es resten les dues equacions resultants.

168

824

824

x

yx

yx

4. Es resol l’equació resultant.

2x

5. Se substitueix el valor de la incògnita trobada en qualsevol de les equacions del

sistema.

se substitueix x = 2 en l’equació 42 yx :

422 y

6. Es resol l’equació resultant.

044 yy

La solució és: x = 2 i y = 0

S’ha de comprovar la solució:

4022

80224

Page 106: TEORÍA DE NÚMEROS1 Principios. Problemas sin teoría. La "Teoría de números" o "aritmética" estudia las propiedades de los números enteros. Los conceptos teóricos de esta rama

Exercici resolt.

Resol el sistema

1232

143

yx

yx

Solució:

Els coeficients de la x són un 3 i un 2. Per tant, multiplicarem la primera per 2 i la

segona per 3, de forma que tots dos coeficients siguin 6:

3696

286

123323

12432

1232

143

yx

yx

yx

yx

yx

yx

Ara resten les dues equacions. Les x desapareixeran, i quedarà una equació en y:

217

3434170

362)98()66(

3696

286

yyx

yx

yx

yx

Trobem el valor de la x substituint el valor de y obtingut a qualsevol de les dues

equacions:

36

18186216621662286 xxxxx

La solució és 2,3 yx

Efectivament:

12662332

1892433

16.1.1 Resol els següents sistemes:

a)

1352

113

yx

yx b)

732

354

yx

yx c)

5835

1372

xy

yx

d)

723

1945

yx

yx e)

1327

2735

yx

yx

16.3.1 Resol els següents sistemes d’equacions, mitjançant el mètode de substitució, aïllant la

incògnita remarcada:

a)

976

153

yx

yx b)

2826

43

yx

yx c)

104

1652

yx

yx

d)

63

52

yx

yx

Page 107: TEORÍA DE NÚMEROS1 Principios. Problemas sin teoría. La "Teoría de números" o "aritmética" estudia las propiedades de los números enteros. Los conceptos teóricos de esta rama

16.3.2

Resol els següents sistemes d’equacions, mitjançant el mètode de substitució:

a)

12)1(2)2(3

103

yx

xy b)

112

43

yx

yx

c)

213

425

yx

yx

d)

4)1(42

24)1(3

yx

yx e)

1132

102)5(4

xy

yx

16.3.3

Resol amb el mètode de substitució:

a)

2xy

xy b)

042

53

yx

yx c)

xyx

xyx

334

65)(3

16.3.4 Sistemes d'equacions per a pensar una mica.

a)

01

4

1

5

04

1

1

3

yx

yx b)

037

8

52

1

023

7

2

5

yx

yx

c)

0132

7

32

5

01

4

1

3

yx

yx d)

31

2

21

4

y

x

y

x

e)

2

2134

115

2

yx

yx

f)

138

234

yx

yx

g)

13

1

4

91

1

3

yx

yx

16.3.5 Repàs de sistemes:

a) Resol per substitució:

112

8

yx

yx b) Resol per igualació:

225

102

yx

yx

c) Resol:

3845

2532

yx

yx d) Resol:

13)1(2

7)1(5)2(3

yx

yx

Page 108: TEORÍA DE NÚMEROS1 Principios. Problemas sin teoría. La "Teoría de números" o "aritmética" estudia las propiedades de los números enteros. Los conceptos teóricos de esta rama

16.3.6 Repàs de sistemes:

a) Resol per substitució:

2332

73

yx

yx

b) Resol per igualació:

145

1832

yx

yx

c) Resol per reducció:

532

1223

yx

yx d) Resol:

13)1(2

7)1(5)2(3

yx

yx

e) Resol:

73

1

53

4

2

3

xy

yx

16.3.7 Repàs de sistemes:

a) Resol per substitució:

42

1055

yx

yx

b) Resol per igualació:

1143

1874

yx

yx

c) Resol per reducció:

222

1042

xy

yx d) Resol:

yx

yx

335

73

26

5

72

Page 109: TEORÍA DE NÚMEROS1 Principios. Problemas sin teoría. La "Teoría de números" o "aritmética" estudia las propiedades de los números enteros. Los conceptos teóricos de esta rama

16.2 Interpretació geomètrica. Mètode gràfic.

Interpretació gràfica d'una equació lineal amb dues incògnites. Una equació lineal amb dues incògnites representa una recta en el pla.

Exercici resolt.

Representa gràficament l'equació 532 yx

Solució:

3

52352532

xyyxyx

(Naturalment, no calia fer una taula de valors tan gran per representar una recta. Amb dos punts n'hi havia

prou!)

16.2.1 Representa gràficament les següents equacions:

a) 123 yx b) 745 yx

c) 132 yx d) 467 yx

Page 110: TEORÍA DE NÚMEROS1 Principios. Problemas sin teoría. La "Teoría de números" o "aritmética" estudia las propiedades de los números enteros. Los conceptos teóricos de esta rama

Interpretació gràfica d'un sistema lineal amb dues incògnites.

Un sistema lineal amb dues incògnites representa la intersecció de dues rectes en el pla.

Exercici resolt.

Interpreta gràficament el sistema

1125

1232

yx

yx

Solució:

Primera recta: 3

21221231232

xyxyyx

Segona recta: 2

11521151125

xyyxyx

Dibuixem les dues rectes al mateix pla. La solució del sistema serà el punt d'intersecció

de les dues rectes:

La solució és 2,3 yx (Comprova-ho resolent el sistema amb un mètode algebraic!)

Page 111: TEORÍA DE NÚMEROS1 Principios. Problemas sin teoría. La "Teoría de números" o "aritmética" estudia las propiedades de los números enteros. Los conceptos teóricos de esta rama

Estudi i interpretació gràfica del nombre de solucions d'un sistema 2x2.

Sistema Compatible Determinat (SCD): Una única solució. Les dues rectes s’intercepten en

un únic punt.

Sistema Compatible Indeterminat (SCI): Infinites solucions. Les dues rectes coincideixen, és

a dir, són la mateixa.

Sistema incompatible (SI): No hi ha cap solució. Les dues rectes són paral·leles.

Classificació dels sistemes d'equacions.

Segons la quantitat de solucions, un sistema pot ser:

Compatible determinat (SCD): Si té una única solució.

Compatible indeterminat (SCI): Si té més d'una solució.

Sistema incompatible (SI): Si el sistema no té solució.

Page 112: TEORÍA DE NÚMEROS1 Principios. Problemas sin teoría. La "Teoría de números" o "aritmética" estudia las propiedades de los números enteros. Los conceptos teóricos de esta rama

Resolució de sistemes 2x2 pel mètode gràfic.

Per "resolució pel mètode gràfic" entenem el dibuixar les gràfiques associades a les dues

equacions i interpretar el seu punt de tall com solució del sistema.

16.2.1 Resol pel mètode gràfic els següents sistemes i observa com queden les rectes de

cadascun.

a)

124

823

yx

yx b)

1

4

xy

yx c)

1462

73

xy

xy

Què observes? El primer sistema és Compatible Determinat (amb solució

2,4 yx ) , el segon sistema és Incompatible i el tercer és un Sistema Compatible

Indeterminat.

Com són les rectes de cada sistema?

Page 113: TEORÍA DE NÚMEROS1 Principios. Problemas sin teoría. La "Teoría de números" o "aritmética" estudia las propiedades de los números enteros. Los conceptos teóricos de esta rama

17 Sistemes d'equacions lineals amb tres incògnites.

17.1 Concepte de sistema d'equacions lineal amb tres incògnites.

Un sistema d'equacions lineals amb tres incògnites té aquesta forma:

3333

2222

1111

dzcybxa

dzcybxa

dzcybxa

La resolució d’un sistema de tres equacions lineals amb tres incògnites es basa en la

resolució pel mètode de reducció o mètode de Gauss. Consisteix a eliminar

adequadament i progressivament incògnites de cadascuna de les equacions per a obtenir

una equació amb una sola incògnita. A partir del valor d’aquesta incògnita, s’aniran

trobant els valors de la resta d’incògnites.

Page 114: TEORÍA DE NÚMEROS1 Principios. Problemas sin teoría. La "Teoría de números" o "aritmética" estudia las propiedades de los números enteros. Los conceptos teóricos de esta rama

17.2 El mètode de Gauss.

El mètode de Gauss és un mètode molt eficaç per resoldre sistemes lineals de tres o més

equacions. Consisteix a anar eliminant les incògnites de cada una de les equacions proposades,

de manera que al final s'obtingui una equació amb totes les incògnites, una altra amb una

incògnita menys, una altra amb dues incògnites menys, i així successivament, fins a l'equació

amb una sola incògnita. El sistema resultant s'anomena sistema triangular. Finalment, es resol

el sistema triangular de manera esglaonada. Aquest mètode també s'anomena mètode de

triangulació.

Exercici resolt.

Resol el següent sistema:

53

4223

934

zyx

zyx

zyx

Solució:

1. Intercanviem, per comoditat, les files 3 i 1.

934

4223

53

53

4223

934

zyx

zyx

zyx

zyx

zyx

zyx

2. A la fila 2 li restem 3 vegades la fila 1. A la fila 3 li restem 4 vegades la fila 1:

111110

111150

53

934

4223

53

zyx

zyx

zyx

zyx

zyx

zyx

3. Intercanviem, per comoditat, les files 2 i 3.

111150

111110

53

111110

111150

53

zyx

zyx

zyx

zyx

zyx

zyx

4. A la fila 3 li restem 5 vegades la fila 2:

444400

111110

53

111150

111110

53

zyx

zyx

zyx

zyx

zyx

zyx

El sistema ja està triangulat. Ara anem aïllant incògnites, de baix a dalt:

Tercera equació: 144

444444444400 zzzyx

Segona equació: 01111111111111110 yyyzyx

Primera equació: 235513053 xxzyx

La solució del sistema és: 1,0,2 zyx

Page 115: TEORÍA DE NÚMEROS1 Principios. Problemas sin teoría. La "Teoría de números" o "aritmética" estudia las propiedades de los números enteros. Los conceptos teóricos de esta rama

17.2.1 Resol els següents sistemes, amb el mètode de Gauss:

a)

34

1223

323

zyx

zyx

zyx

b)

93

1

92

zyx

zyx

zyx

c)

11296

5534

662

zyx

zyx

zyx

d)

28312

18494

2638

zyx

zyx

zyx

Page 116: TEORÍA DE NÚMEROS1 Principios. Problemas sin teoría. La "Teoría de números" o "aritmética" estudia las propiedades de los números enteros. Los conceptos teóricos de esta rama

18 Sistemes d'equacions de segon grau.

18.1 Resolució de sistemes d'equacions de segon grau.

Els sistemes d'equacions no lineals es resolen, en general, amb el mètode de substitució.

Exercici resolt.

Resol el sistema

096

02

22 yx

yx

Solució:

Aïllem la y de la primera equació: xyyx 202

La substituïm a la segona equació: 09260962222 xxyx

Resolem l'equació en x resultant:

5

3

25

9

25

9

925092509240946

2

222222

xx

xxxxxx

Substituïm els valors obtinguts per obtenir

5

6

5

32

5

3

5

6

5

32

5

3

2

yx

yx

xy

Les solucions són: 5/6,5/3 yx i 5/6,5/3 yx

18.1.1 Resol els següents sistemes de segon grau:

a)

193

3

2x

y b)

8

2

2 xy

x c)

72

82

yx

yx

d)

43

632

yyx

yx e)

2

23 2

yx

yx f)

03

163)2(4

2

22 xyx

yx

18.1.2 Resol els següents sistemes quadràtics:

a)

172

10

22

22

yx

yx b)

29

7

22 yx

yx c)

6

1

xy

xy

d)

574

102

2 xyx

yx e)

6

14

2

2

xyx

xyx f)

5

20

y

x

xy

Page 117: TEORÍA DE NÚMEROS1 Principios. Problemas sin teoría. La "Teoría de números" o "aritmética" estudia las propiedades de los números enteros. Los conceptos teóricos de esta rama

18.1.3 Resol els següents sistemes:

a)

1076

1635

2 yx

yx b)

954

12

22 yx

yx c)

8

2034

xy

yx

18.1.4

Resol els següents sistemes:

a)

255

2

22 xyyx

yx b)

2632

223

2yxyx

yx

c)

52

1432

yx

yyxxyx d)

1832

2

2yx

yx

e)

42

1322

yx

yx f)

13

2

111

yx

yx

g)

3

21323122

yx

yxx h)

90

18922

yx

yx

i)

84

8

yx

yx

Page 118: TEORÍA DE NÚMEROS1 Principios. Problemas sin teoría. La "Teoría de números" o "aritmética" estudia las propiedades de los números enteros. Los conceptos teóricos de esta rama

19 Sistemes d'equacions racionals.

Exercici resolt.

Resol el sistema

022

12

2

2

yx

yy

x

Solució:

Simplifiquem la primera equació:

yxxyyyxyy

x242)2(2)12(

12

2

2

El sistema que obtenim és:

022

242

yx

yxxy

Apliquem el mètode de substitució: Aïllem la y de la segona equació:

xyyx 22022

i la substituïm a la primera:

4

141041

0

0)41(04

044344344444

44444)22(24)22(2

2

222

2

xxx

x

xxxx

xxxxxxxxxx

xxxxxxxx

Amb els valors de x trobats trobem els valors de y:

20220 yx

2

5

4

122

4

1

yx

Comprovem les solucions obtingudes al sistema inicial:

02202

122

2

22

0

2,0 yx

Veiem que aquesta solució està fora del domini de definició de la primera equació, i per

tant no és vàlida.

00

4

2

2

1

02)2/5()4/1(2

1)2/5(2

2

2/52

4/1

2

5,

4

1yx

Aquesta solució és vàlida, per tant la única solució del sistema és 2

5,

4

1

yx .

19.1.1

Page 119: TEORÍA DE NÚMEROS1 Principios. Problemas sin teoría. La "Teoría de números" o "aritmética" estudia las propiedades de los números enteros. Los conceptos teóricos de esta rama

Resol els següents sistemes d'equacions racionals:

a)

5

3

10

2

5

2

5

2

464

yx

yx b)

yx

yy

x

y

x

3

162

c)

1

4

1

2

1

)1()2(23 222

xx

x

x

y

yyxyxx

d)

52123

)3(2

13

6

)1(11

)3(

12

yxx

x

x

y

x

xy

x

e)

x

y

xxx

x

xy

26

6

3

2

3

2

f)

12

422

1

1

)2(423

y

y

x

x

yxy

Page 120: TEORÍA DE NÚMEROS1 Principios. Problemas sin teoría. La "Teoría de números" o "aritmética" estudia las propiedades de los números enteros. Los conceptos teóricos de esta rama

20 Sistemes d'inequacions amb una incògnita.

Un sistema d’inequacions amb una única incògnita està format per diverses inequacions i

limitat per una clau que indica precisament que es tracta d’un sistema, i no d’equacions

independents. Per exemple, un sistema d’inequacions podria ser:

12

283

2 xx

x

Un nombre és solució d’un sistema d’inequacions d’aquest tipus si és solució de totes les

inequacions que formen el sistema. Per exemple, x = 1 és una solució del sistema

d’inequacions:

11121

211813

2

Per trobar les solucions d'un sistema d'inequacions resoldrem cada inequació per separat

i determinem la intersecció dels intervals obtinguts.

20.1 Sistemes d'inequacions de primer grau amb una incògnita.

Exercici resolt.

Resol el següent sistema d'inequacions de primer grau:

042

093

x

x

Solució:

Resolem la primera inequació: 093 x

Punt frontera: 33/9093 xx

Punt de prova: 0909030 x

Interval solució: 3,

Resolem la segona inequació: 042 x

Punt frontera: 22/4042 xx

Punt de prova: 0404020 x

Interval solució: ,2

Trobem la intersecció dels dos intervals solució: 3,2,23,

La solució del sistema és l'interval 3,2

Exercici resolt.

Page 121: TEORÍA DE NÚMEROS1 Principios. Problemas sin teoría. La "Teoría de números" o "aritmética" estudia las propiedades de los números enteros. Los conceptos teóricos de esta rama

Resol el sistema d'inequacions:

2562

12)1(3

xx

xx

Solució:

Resolem la primera inequació: ,5/212)1(3 xx

Resolem la primera inequació: 3/4,2562 xx

La intersecció dels dos intervals és el conjunt buit: ,5/23/4,

Per tant el sistema no té cap solució, o dit d'una altra manera, el conjunt solució és el

conjunt buit.

20.1.1 Resol els següents sistemes d'inequacions:

a)

3)2(32

7)2(3)1(2

2

2

xxxxx

xxxxxx

b)

732

23)1(37 2

x

xxxxxx

c)

2874

176)3(17 2

xx

xxxxx d)

236

3)1(2 22

xx

xxxx

e)

xxxxx

xxx

9)2(717

172)1(

2

22

f)

162

1)2(216 22

xx

xxxxx

g)

312

2)1(6 2

x

xxxx h)

22

22

1233)2(

)2(236

xxxx

xxxxx

i)

313

72)2)(12( 2

xx

xxxx j)

2

22

4)1(2

27)3(

xxxx

xxx

Page 122: TEORÍA DE NÚMEROS1 Principios. Problemas sin teoría. La "Teoría de números" o "aritmética" estudia las propiedades de los números enteros. Los conceptos teóricos de esta rama

20.2 Sistemes d'inequacions de segon grau amb una incògnita.

Exercici resolt.

Resol el següent sistema d'inequacions de segon grau:

0152

045

2

2

xx

xx

Solució:

Resolem la primera inequació: 0452 xx

Punts frontera: 4,10452 xxxx

Punt de prova: 04040500 2 x

Interval solució: 4,1

Resolem la segona inequació: 01522 xx

Punts frontera: 3,501522 xxxx

Punt de prova: 0150150200 2 x

Interval solució: 3,5

Trobem la intersecció dels dos intervals solució: 3,13,54,1

La solució del sistema és l'interval 3,1

20.2.1 Sistemes "Primer Grau & Segon Grau":

a)

01

022

x

x b)

03

022

x

xx c)

027308

023

2 xx

x

d)

02

013

2x

x

e)

032

037

012

2

x

xx

x

f)

0169

01

074

2x

x

x

g)

016

013

032

2 xx

x

x

Page 123: TEORÍA DE NÚMEROS1 Principios. Problemas sin teoría. La "Teoría de números" o "aritmética" estudia las propiedades de los números enteros. Los conceptos teóricos de esta rama

20.2.2 Resol els següents sistemes d'inequacions:

a)

02

032

045

2

2

x

xx

xx

b)

072

082

0352

2

2

2

xx

xx

xx

c)

027394

07132

01892

2

2

2

xx

xx

xx

d)

02

0427

0576

2

2

2

xx

xx

xx

e)

012112

03116

09124

2

2

2

xx

xx

xx

Page 124: TEORÍA DE NÚMEROS1 Principios. Problemas sin teoría. La "Teoría de números" o "aritmética" estudia las propiedades de los números enteros. Los conceptos teóricos de esta rama

Solucions

1.1.1 a) racional b) racional c) enter d) racional e) natural

f) racional g) irracional h) natural i) enter j) irracional

k) natural l) enter m) natural n) racional

1.1.2

1.1.3

2.1.1

2.1.2

2.1.3

2.1.4

2.1.5

2.2.1

2.2.2 a) 2/310 b) 4/55 c) 3/12

2.2.3 a) 3 b) 10 c) 216

3.1.1 a) ,5 b) 5.4,0 c) 5.2,1 d) 4,3 e) 5.2,1

3.1.2 a) 3,3 b) 3,5 c) ,33, d) ,104,

3.1.3

3.1.4

3.1.5

3.1.6

3.2.1 a) 11,7 b) 15,1 c) 2,2 d) 1,5

3.2.2

4.1.1

4.1.2

4.1.3

4.1.4

4.1.5

4.2.5 a) 54m b) m2 c)x

8 d) n8 e) 58k f) 24x g) xy26 h) 244 uv i)

ab

12

j) 2

5

y

x

4.2.6 a) 1 b) 816

1

x c) 256 d) 616a e) 1681k f)

xy4

1

g) 42

1

b h)

2

4

y

x i)

4

3

2x

y j)

29

1

m k)

x2

1 l)

54

1

x

m) n n) 2

1 o)

7

3

m p)

7

2

3

2

yz

x q)

xy

z2

3

r) 4

33

3

2

j

kh s)

p

nm

3

4 2

t) yz

x73

4.2.7 12

1

5.1.1

5.2.1 a) 7

5.2.2 a) 2/310

Page 125: TEORÍA DE NÚMEROS1 Principios. Problemas sin teoría. La "Teoría de números" o "aritmética" estudia las propiedades de los números enteros. Los conceptos teóricos de esta rama

5.2.3 a) 3

5.3.1 a) 2 b) 5 4 c) 5 d) 2 e) 6 f) 5 3

g) 2 h) 3 i) 3 5 j) 5 5 k) 3 l) 5 7

5.3.2 a) 3

5.3.3

5.3.4 a) 34

5.3.5

5.4.1 a) 666 5,27,4 b) 121212 2401,216,52 c) 666 30,36,125

d) 101010 60,3125,100

5.4.2 a)

5.5.1 a) 253 b) 77 c) 23 d) 0

5.5.2 a) a3 b) 0 c) a d) 0 e) ba4

1

4

7

f) bab 9 g) y h) ba )6/7(

5.5.3 a) 317

5.5.4 a) 85 b) 32 c) 26 d) 2232 e) 6255 f) 63

g) 613 h) 236 i) 6332

5.5.5 a) 242 b) 52 c) 37 d) 224 e) 23064

f) 54621 g) 74217

5.6.1

5.6.2 a) 218 b) 25 c) 40 d) 512

5.6.3 a) 10

3 b)

5

2 c)

3

2 d)

2

53

5.7.1 a) 335 b) 54302 c) 2936 d) 23305

5.7.2 a) 591515 b) 5915 c) 51126 d) 15418

5.7.3 a) 4 b) -23 c) -7 d) -11 e) -2 f) -5

5.7.4 a) 16312 b) 30513 c) 2 d) 31727

5.8.1 a) 5

54 b)

15

32 c)

3

15 d)

6

6

5.8.2 a)25

154 b)

6

6

5.8.3 a) -7 b) 3 c) 15 d) -17

5.8.5 a) 16

353 b)

2

25315 c)

11

1757 d)

4

6535222

e) 11

10228554

5.8.6 a) 3

324 b)

12

34 c)

5

32 d)

4

13

e) 52

655132 f)

68

17485 g)

3

96 h)

18

3230

Page 126: TEORÍA DE NÚMEROS1 Principios. Problemas sin teoría. La "Teoría de números" o "aritmética" estudia las propiedades de los números enteros. Los conceptos teóricos de esta rama

i) 43

25515 j)

77

52035 k)

23

2210 l)

2

232

m) 11

3912 n) 422 o) 53 p)

2

35 q) 21 r)

43

54316 s) 12 t) 323 u) 2 v) 2

5.8.7 a) a b) a c) ba

abba

d)

ab

abba

5.8.8 a) y

xyx 3 b)

y

y

2

6 c) 492142 xx

5.8.9 a) 12 b) 2

17 c) 16 d) 15 e) 25

f) 135 g) 35 h) 236 i) 152 j) 41

227

k) 3

323

5.8.10 a) x

x

31

322

b)

x

xx 462 c)

3

62

m

m

x.x.x a)

3

522 b)

73

32024 c)

11

53 d)

23

252

7.1.1

7.1.2 a) 71086.7 b) 31094.3 c) 0107.4 d) 61026.1 e) 2106

f) 21075.1

7.1.3 a) 6170 b) 70000 c) 7310000 d) 0.000000054

e) 0.0067 f) 959

7.1.4 a) 5102 b) 7103 c) 41084.8 d) 81088.2

7.1.5 a) 6106 b) 6104.5 c) 1106 d) 7102

e) 6102 f) 4101.7 g) 7109 h) 0103.6 i) 41016.2

j) 3102.4

7.1.6 a) 0.09 b) 0.2 c) 20000 d) 80400 e) 26600

f) 0.015 g) 0.775 h) 83000000 i) 4 j) 0.00004

8.1.1 a) -6 b) 4 c) 5 d) 4 e) -8 f) -8 g) 7 h) 6 i) j) -1 k) -1 l) 3

m) 3 n) 1 o) 1 p) 6 q) IR r) -11 s) t) 7

8.1.2 a) 7 b) 4 c) 6 d) 3 e) 1 f) 6 g) 2 h) 1 i) 11 j) 11 k) 4

l) 3 m) 1 n) 5

8.1.3 a) 12 b) 4 c) -2 d) 1 e) 3 f) 5 g) 6 h) 8 i) 2 j) -1 k) 9 l) -29

m) 13 n) 1

8.1.4 a) -1/3 b) 52

3

c)

3

1 d) 3 e) 0 f)

71

7

9.1.1 a) 18, 0 b) -1 , 12

9.1.2 a) -12 b) 0 c) 42 d) 5 e) -20 f) -1/4 g) -7/3 h) 0

9.1.3 a) 32 , 2 , -1/2 b) 112 , -20 , -9 c) -21/2 , 0 d) -96 , 7/48 , 0

e) -7 , 1 f) -4 , -5/2 , -4

9.1.4 a) -7/9 b) 26 c) 5 d) 9 e) 10

Page 127: TEORÍA DE NÚMEROS1 Principios. Problemas sin teoría. La "Teoría de números" o "aritmética" estudia las propiedades de los números enteros. Los conceptos teóricos de esta rama

9.2.1 a) 57, 52 b) 59, -13 c) -2, -4

9.3.1 24363 234 xxxx , 661583 234 xxxx

9.3.2 a) 153 a b) 1224 a c) 3322 a d) 21140 b e) x24

f) 23 12aa g) aaa 21183 23 h) 23 2826 xx i) 1245 x

j) 1200018000 x k) aaa 552211 23 l) 9000240 x

9.3.3 a) 32012 2 xx b) 82620 2 xx c) 3852 23 xxx

d) 810135 23 xxx e) 5424144 23 xxx

9.3.4 a) 3586 23 xxx b) 482062 23 xxx

c) 1529226 23 xxx d) 142533 23 xxx

e) 5492115 23 xxx

9.3.5 a) 3563 234 xxxx b) 214733142 234 xxxx

c) 104633192 234 xxxx d) 24215196 234 xxxx

9.3.6 a) 345 3 xxx b) 5678 4614104 xxxx c) 1623224 234 xxx

d) 43192196 2345 xxxxx

e) 72173617852 2345678 xxxxxxxx

f) xxxxx 20729192 2345 g) 32529102 234 xxxx

h) 5342782 234 xxxx

9.4.1 a) 169 2 xx b) 9124 24 xx c) 456 446 yyy

d) 4/124 aa

9.4.2 a) 8126 23 xxx b) 133 246 xxx

9.4.3 a) 122 xx b) 442 xx c) 144 2 xx

d) 4129 2 xx e) 12 36 xx f) 168 24 xx

9.4.4 a) 23x b) 25x c) 24x d) 27x e) 232 x

f) 215 x g) 2yx h) 224 x i) 223 yx

9.4.5

9.4.6 a) 122 xx b) 442 xx c) 169 2 xx

d) 9124 2 xx e) 12 24 xx f) 96 36 xx

9.4.7 a) 23x b) 252 x c) 212)4/1( x d) 224 x

e) 252 x f) 273 x g) 275 x h) 234 x i) 252 x

9.4.9 a) 116 6 x b) 464 ba

9.4.10 a) )52)(52( xx b) baba 4343 22 c) 4545 xx

d) yxyx 5353

9.4.11 a) 12 24 xx b) 144 2 xx

9.5.1 a) Q: 3x , R: 513 x b) Q: 2x , R: 0 c) Q: 1x , R: 0

d) Q: 432 xx , R: 0

9.5.2 a) Q: 1x , R: -2 b) Q: 231252 23 xxx , R: 7261 x

c) Q: 1234567 xxxxxxx , R: 0

9.5.3 a) Q: 123 xxx , R: 33 x b) Q: 22 x , R: 12 x

c) Q: 33 36912 xxxx , R: -3

9.5.4 a) Q: 5432 3468 xxxx , R: 6 b) Q: 12468 xxxx , R:0

c) Q: xxxx 214161 , R: x

9.5.5

9.6.1 a) Q: 125 23 xxx , R: -2 b) Q: 4355 234 xxxx , R: 4

Page 128: TEORÍA DE NÚMEROS1 Principios. Problemas sin teoría. La "Teoría de números" o "aritmética" estudia las propiedades de los números enteros. Los conceptos teóricos de esta rama

c) Q: 126 xxx , R: -5

9.6.2 a) Q: 82 2 xx , R: -1 b) Q: 52148526 2345 xxxxx , R: 69

c) Q: 9

37

3

82 xx , R: 27

17

9.6.3 a) Q: 122 xx , R: 3 b) Q: 232 2 xx , R:1

c) Q: 372 245 xxxx , R: 7 d) Q: 134 235 xxx , R: 7

9.6.4

9.6.5

10.1.1 a) 5 b) 5 c) a2 d) a4 e) 3 f) 4 g) 35x h) 32x i) 48y j) xy2 k) 5b l) cb33

10.1.2 a) 56611 cba b) 65)16/1( yx c) )5(2 a d) 3)(4 yx e) )4()3(7 4 aa

f) 53 )3()5(13 yxyx

10.2.1 a) )52(2 x b) )3(6 x c) )5(5 x d) )3(3 aa e) )87(2 bb

f) )43(7 xx g) )109(5 yy h) )29(2 aa i) )35(8 2 yy

j) )34(4 2 xx

10.2.2 a) ))(4( bay b) )4)(34(2 2 nmm c) ))(9( bax

10.2.3 a) )32)(9( ba b) )7)(4( yx c) )3)(1( xy

d) )4)(5( rsab e) )37)(32( bayx f) )2)(2(3 yxmb

g) )2)(3( 22 ba

10.2.4 a) )2)(13( 2222 baabba b) )53)(2( 22 zywxzxy

c) ))(( 2 xyabcdcxba

10.2.5 a) )2(3 yx b) )( 22 yaxa c) )7(2 cba d) )124(2 22 aaa

e) )32)((3 yxyx f) )49(3 23 zyzyz g) )3(9

1 2

xyyx

h) )34(5 22 yxxx i) )65(5 22 baba

10.2.6 a) ))(( bayx b) )31)(3( yxyx c) )1)(( xba

d) )2)(1(2 yxa e) )1)(( baba f) )321()32( 222 yxyx

g) 1 a h) )144)((2

1 abyx i) )1)(( azyxa

j)

2)3(

2

12)3(

3

1baba k) )26)(6(

4

1 yxyx

10.3.1 a) )5)(5( mm b) )32)(32(9 qpqp c) )7)(7( 22 bcabca

d) )10)(10( 246246 yxwyxw e) )5)(5(3 xx

f) ))(( 22 mnabmnabam

10.3.2 a) ))()(( 22 nmnmnm b) )221)(221)(12)(12( 2222 yyyyyy

10.4.1 a) 2)13( x b) 2)2( ba c) 2)32( b d) 233 ba e)

2

3

1

xa

f) 212 a g)

2

2

13

y h) i) 22 )2(3 ax j)

2

2

1

ba

10.4.2 a)

2

2

5

5

2

y b) 22 8 x c) d) 2)7( baa

Page 129: TEORÍA DE NÚMEROS1 Principios. Problemas sin teoría. La "Teoría de números" o "aritmética" estudia las propiedades de los números enteros. Los conceptos teóricos de esta rama

e) 2234 xyyx f)

2

2

13

xba g)

2

2

3

3

2

ab h) 21x

i)

2

4

12

yx

10.5.1 a) )12)(2)(1( aaa b) )43)(1( 2 bbb

c) )4242)(2( 234 xxxxx d) )4)(3)(2)(2( 2 aaaa

e) )12)(3)(1( bbb f) )13)(12)(1( xxxx

10.5.4 a) )3)(5(2 xx b) 2)12( x c) )13)(1( xx d) )1)(12( xx

e) )13)(12( xx

10.5.5 a) )2)(1)(4( xxx b) )1)(2)(3( xxx

c) )3)(1)(3(2 xxx d) )53()1( 2 xx

10.5.6 a) )5)(3)(1)(2( xxxx b) )3)(3(3 xxx c) )1)(3(2 xxx d) 22 )3()1( xx e) )2)(1)(2)(4(2 xxxxx

10.5.7 a) -3, 2 b) -1, 2, 3

10.5.8 a) -1, 0 b) 0, 8 c) -1, 3 , 5

10.6.1 a) -3 , 1 , 2 b) -2 c) 1 , 3 d) -3, -1 , 2 e) -1 , 1 , 2

10.6.2 a) -1/2 , 2/3 , 1 b) -1 , 1/5 , 3/2 c) -1/3 , 1/2 , 1 d) -1 , 2, 1/2

e) 2/3 f) -3/2 , -1 g)1/4

11.1.1 a) 4, 8 b) 1, 6 c) -5, -4 d) -6, -1/2 e) 3, 6/5 f) -1/5,

3/2 g) -5/6, 6/11 h) 2/7 i) 6/5

11.2.1 a) 1 b) 2 c) d) 3/6 e) 6 f) g) 1

11.2.2 a) 4 3 b) 1 c) 63 d) 4 2/1 d) e) 3/2

11.3.1 a) 0 , 7/2 b) 0 , -6 c) 0 , -3 d) 0 , -2 e) 0 , -5

11.3.2 a) 0 , 323 b) 0 , 23 c) 0 , 3/3 d) 0 , 6

e) 0 , 2/32 f) 0 , 32 g) 0 , 12

11.3.3

11.3.4

11.3.5 a) -11, 0

11.4.6

11.3.6 a) 3/4 , 0

11.3.7

11.4.1 a) -1 , 3 b) -5 , 2 c) d) -4 , 8 e) 2/3

11.4.3 a) 1 , 5

11.4.4 a) -4/5 , 2

11.5.1 a) -2 , 4 b) -6 , 10 c) 4 , 10 d) -5 , 10 e) -8 , 1

f) -5/2 , 3 g) -3 , 2/3 h) 1/2 i) 1 , -9 j) 13 , -1

k)

11.5.2 a) 4 b) -6 , 8 c) 21 d) 1 , -5 e) 336

f) -1 , -1/4 g) 524 h) i) 51 j) k) 22

11.5.3 a) -3 , -2 b) -4 , -2 c) 7 , -2 d) 5 , -2 e) 3 , -1

11.5.4 a) -1/2 , 6

11.5.5 a) -3/2 , 2

11.7.1 a) )2( 22 uu b) )4)(3( 22 xx c) )5)(1( 22 aa

Page 130: TEORÍA DE NÚMEROS1 Principios. Problemas sin teoría. La "Teoría de números" o "aritmética" estudia las propiedades de los números enteros. Los conceptos teóricos de esta rama

d) )5)(3( 22 xx e) )1)(5( 22 uu f) )5)(4( 22 mm

g) )1)(3( 22 xx h) )5)(2( 22 xx i) )8)(27( 22 mm

j) )5)(67( 22 uu k) )1)(1)(45( 2 xxx l) )2)(43( 22 xxx

m) )6)(12( 222 xxx n) no factoritzable

o) )6)(27( 22 mm p) )2)(73( 22 uu

q) )1)(1)(42)(2( 22 xxxxxx r) )5)(10(6 44 xxxn

s) )10)(6( 33 xx t) )83(5 48 uun u) )3)(1)(1( 32 xxxx

v) )9)(9( 33 mm w) )3)(5( 33 xx x) )5)(3( 33 xxxm

12.1.1 a) No està definida si 2x b) No està definida si 2

51x

c) Sempre està definida.

12.2.1 a) 7

6 b)

3

3

x

x c)

4

1 d) 2226 cba e)

8

1

x

x f) 1 g)

)9(2

)5(3

a

aa

h) 2)10( x

12.2.2 a) 1

4

x

x b)

3

2

x

x c)

1

3

x

x d)

x

x 4 e)

3

12

b

b

f) yx

yx

2

910

12.2.3 a) 13 x b) 32 x c) 22 x d) 5x e) 4x

f) 2x

12.2.4 a) 3,1,1

1

bb

b b) 0,

2

2

aa

c) 5,1,5

2

xx

x

x

d) 0,5

xxy

e) 0,0,9

2 yx

x

yz

f) 0,0,0,3

zyxx

g) 3,0,3

8

aa

a h) 2,

2

4

x

x

x

i) 0,2

510

x

x

x j) 5,5 xx k) 2,

4

2

b

b

l) 1,1,1

3

xx

x m) 2,7,

2

1

aa

a n) 4,

6

2

x

x

o) 1,6,1

1

yy

y

y

12.3.1 a) 2y

x b)

yx

yx

43 c)

103

133 2

x

xx d)

)23(

74 2

xx

x

e) )2)(3(

47

xx

x f)

)6(

124 2

aa

aa g)

)4)(2(

35 2

xx

xx

12.3.2 a) )1)(2(

)78(

bb

bb b)

)3)(2(

2542 2

aa

aa c)

)3)(3(

18193 2

xx

xx

d) yy

yy

)4(

1265 2

e)

)6)(1(

2

yy

y f)

)3)(3(

37 2

aa

aa

Page 131: TEORÍA DE NÚMEROS1 Principios. Problemas sin teoría. La "Teoría de números" o "aritmética" estudia las propiedades de los números enteros. Los conceptos teóricos de esta rama

g) )4)(2)(1(

)52(6

aaa

a h)

22

2

)3()2(

)362(2

bb

bb i)

)4)(1(3

852 2

xx

xx

j) 4

12

x

x

12.3.3 a) )2)(1(

35

xx

x b)

)3)(4)(5(

2932 2

bbb

bb c)

)3)(1(

97

aa

a

d) )8)(6)(2(

)13(2

yyy

y e)

)4)(3(

)34( 2

yy

yyy f)

)7)(2)(2(

29

xxx

x

12.3.3 a) )1)(2)(3(

15

xxx

x b)

)4)(2)(2)(3(

1431223 23

xxxx

xxx

c) )1)(1)(4(

8102

yyy

yy d)

)3)(2(4

18624

23

xxx

xxx e)

)2)(3)(6(

)2)(5(

aaa

aa

f) )1)(6(

22 2

yyy

yy g)

)3)(3(4

)37)(1(

xxx

xx

12.3.4 a) 6

27132 2

y

yy b)

6

458

x

x c)

2

)6(2

y

y d)

7

3

x

x

e) 6

133

x

x f)

1

423 2

x

xx g)

3

572

y

yy h)

1

22 2

x

xx i)

2

)1)(7(

b

bb

12.3.5 a) m

1 b)

3

x c)

x

6 d)

2

1 e)

2

35 y f) x2

g) 1 h) 2k

k i) 1 j)

32

62

n

n k) 12 d

l) 4

8

a m)

5

1 n) 1 o)

2

3

x

x

12.3.6 a) x10

3 b)

x7

15 c)

2

)2(5

xy

yx d)

3

279

a

a e)

63

17

x

f) )4(2

3

x g)

x

x

44

9

12.3.7 a) )6)(3(

338

xx

x b)

)3)(3(

12115 2

xx

xx c)

)4)(4(

89

xx

x

d) )10)(10(

3102

xx

xx e)

)6)(1(

)423(

xx

xx

12.5.1 a) 1/3 b) -1/2 c) 2/11 d) -3 e) 2

12.5.2 a) 4 b) 8

12.6.1 a) -2 b) 2 c) -10 d) 1

12.6.2 a) 1/6 b) -1/2 c) 5 d) 4 e) -1/5 f) 21

g) 5 h) 2 i) 26/5 j) 36/7 k) -5/4 l) -3 , 6

m) 1 n) -17/3 o) 4 , -1 p) 36/7 q) -19/8 r) 4 , 1

s) -1/4

12.6.3 a) 14

12.6.4 a) -1

Page 132: TEORÍA DE NÚMEROS1 Principios. Problemas sin teoría. La "Teoría de números" o "aritmética" estudia las propiedades de los números enteros. Los conceptos teóricos de esta rama

13.1.1 a) 5x b) 8y c) 3/2a d) 5/6m e) 4x

13.2.1 a) 7 b) 15 c) 43 d) 2 , 6 e) 3 f) -2 g) 1

13.2.2 a) 3 b) 1/2 c) 3 d) 2 e) 5 f) 7 g) 5 h) 5

13.2.3

13.2.4 a) 14

13.2.5 a) 1

13.2.6 a) 5

13.2.7 a) 5

13.3.1 a) 11/30 b) 69434 c) 10/3 d) 32/323 e) 1 f)

g) 0 , 1/4 h) 0 , -2/5 , 2/5

13.3.2 a) 4

13.3.4 a) -2 , 2

x.x.x a) 3/4 b) c) d) -1/2 e) -3/8, 1/9

f)1/4 g) -3/4 , -5/7

14.2.1 a) -3/2 b) 3 c) -2 d) 0 e) 2/3

f) -1/4 g) 2 h) 2

14.2.2 a) 0 b) 0 c) 1/2 d) -2/3 e) -3 e) 1 f) 10

g) 0 h) 7/12 i) -4/17 j) -3/4 k) -3/4 l) -1/6 m) -2/3

n) 6/7 o) 4

14.4.1 a) 1 b) 2 c) 1 d) 0

14.4.2 a) 0 b) 0, 0.6931 c) 4 , 2 d) 1.60944 e) 1 , 2 f) 2, 1

g) 0 h) 6.41504 i) 4 j) 0 k) 1 l) 1 m) -2

n) 2, -2 o) 6 p) 0.5

14.5.1 a) 243 b) 64 c) 50 d) 3 e) 4 f) 35 g) 102/5

14.5.2 a) 3 b) 5

14.6.1 a) 2 b) -1 c) 0 d) 16 e) 4

14.7.1 a) 28/3 b) 53/5 c) 3 d) e) 11

f) 1/6 g) h) 33/7

14.7.2 a) 27/2 b) 25

14.7.3 a) 6 b) 8

14.7.4 a) 103/5 b) 4 c) 2 d) 3/4 e) 7 f) 5

14.7.1 a) 3

52 b) 0 c) 2 d)

2

32 e) 2 , 3 f) 3, 1/3

g) 1, 9 h) 4 i) 6 j) 8 k) 12/5

14.8.2 a) 2/3 b) 4 c) 1 d) 0.861353 e) 0 i 1 f) –1 g) 0 i 2 h) 1

i) –1 i 1

14.8.3 a) 1.86191 b) 4 c) 2 d) 3/4 e) 7 f) 5

x.x.x a) 0.792 b) 9 c) 0.252 d) 2.087

e) -0.251 , -1.999 f) -0.25 g) 1.096 , -1.596 h) 0.375

i) -0.35 j) 0.957

15.1.1 a) 3x b) 4/7x c) 3/2x d) 4/3x e) 2x

15.1.2 a) 2x b) 4x c) 2/5x d) 2/1x e) 19x

f) 4x g) 7x h) 2x i) 14x j) 11x

15.1.3 a) 4x b) 3/7x c) 2/5y d) 9x e) 3x

f) 3a g) 0x h) 1x i) 1x j) 4w

k) 5x l) 0x m) 5x n) 8x o) 12x

p) 13x q) 12x r) 5x

Page 133: TEORÍA DE NÚMEROS1 Principios. Problemas sin teoría. La "Teoría de números" o "aritmética" estudia las propiedades de los números enteros. Los conceptos teóricos de esta rama

15.1.4 a) ,9/2 b) ,19/1 c) , d) ,1

e) 2, f) 1,

15.1.5 a) 3

4x b)

5

13x c)

4

15x d)

7

10x

15.1.6 a) 5

12x b) 7x c)

10

19x d)

3

2x

15.2.1 a) 32 x b) ,31,

15.2.4 a) 8,2 b) ,43, c) 5,2

15.2.5 a) ,2/91, b) 142,142 c) 12,1

d) ,221, e) ,2424,

15.2.6 a) ,12, b) ,23, c) 3/1,2/1

d) ,42, e) 2,1 f) ,31,

g) 2,4 h) ,2/54/3, i) 2/3,3/2

15.2.7 a) ,12/1, b) ,5/35/2, c) 3/1,3/4

d) ,2/11, e) 4/3,8

f) ,34,

15.2.8 a) , b) ,3/23/2, c) 9/1

d) , e) f) g) ,4/94/9,

h) i) , j) ,3/53/5,

15.3.1 a) 3,12

1,

b)

,1

3

2,1 c) ,1

d)

,1

2

1,1 e) 5,13, f) ,3/5

15.3.2 a) 1,2/11, b) ,5/22/1,1

15.3.3 a) ,43/1,3/14, b) 5,3/33/3,5

15.3.4 a) 3,2IR b) ,12/1, c) 2,4/34/3,2